*NURSING > HESI > NURS 203HESI Final Questions and answers Graded A (All)

NURS 203HESI Final Questions and answers Graded A

Document Content and Description Below

NURS 203HESI Final 1- A client with multiple sclerosis is receiving beta – 1b interferon every other day. To assess for possible bone marrow suppression caused by the medication, which serum lab... oratory test findings should the nurse monitor? (Select all that apply) a- Platelet count b- White blood cell count (WBC) c- Sodium and potassium d- Red blood cell count (RBC) e- Albumin and protein 2- A male client with hypercholesterolemia wants to change his diet to help reduce his cholesterol levels. When breakfast items should the nurse encourage the client to eat? (Select all that apply) a- Sausage patties and eggs b- Whole wheat toast and jam c- Bagels and cream cheese d- Toaster pastries and milk e- Blackberries and oatmeal 3- After an unsuccessful resuscitation attempt, the nurse calls the family of the deceased. The family wish to see the body before it is taken to the funeral home. Which interventions should the nurse take to prepare the body before the family enters the room? (Select all that apply) a- Take out dentures and place in a labeled cup b- Apply a body shroud c- Place a small pillow under the head d- Remove resuscitation equipment from the room e- Gently close the eyes 4- A client with major depression who is taking fluoxetine calls the psychiatric clinic reporting being more agitated, irritable, and anxious than usual. Which intervention should the nurse implement? a- Tell the client to have a complete blood count (CBC) drawn b- Instruct the client to seek medical attention immediately c- Encourage him to take the medication at night with a snack d- Explain that these are common side effects of the medication 5- An older adult male is admitted with complications related to chronic obstructive Pulmonary Disease (COPD). He reports progressive dyspnea that worsens on exertion and his weakness has increased over the past month. The nurse notes that he has dependent edema in both lower legs. Based on these assessment findings, which dietary instruction should the nurse provide? a- Restrict daily fluid intake b- Eat meals at the same time daily c- Maintain a low protein diet d- Limit the intake of the high calorie foods 6- A client with diabetic peripheral neuropathy has been taking pregabalin for 4 days. Which finding indicates to the nurse that the medication is effective? a- Granulating tissue in foot ulcer b- Full volume of pedal pulse c- Reduced level of pain d- Improved visual activity. 7- The nurse is assessing an older adult with type 2 diabetes. Which assessment finding indicates that the client understands long- term control of diabetes? a- The fating blood sugar was 120 mg/dl this morning. b- Urine ketones have been negative for the past 6 months c- The hemoglobin A1C was 6.5g/100 ml last week d- No diabetic ketoacidosis has occurred in 6 months. 8- A heparin infusion is prescribed for a client who weights 220 pounds. After administering a bolus dose of 80 units/kg. The nurse calculates the infusion rate for the heparin sodium at 18 units/kg/hour. The available solution is Heparin Sodium 25,000 units in 5% Dextrose Injection 250 ml. The nurse should program the infusion pump to deliver how many ml/hour. (Enter numeric value only. If rounding to the nearest whole number.) 18 9- The nurse is assessing a client with Addison's disease who is weak, dizzy, disoriented, and has dry oral mucous membranes, poor skin turgor, and sunken eyes. Vital signs are blood pressure 94/44, heart rate 123 beats/minute, respiration 22 breaths/minute. Which intervention should the nurse implement first? a- Assess extremity strength and resistance b- Report a sodium level of 132 mEq/L or mmol/L (SI units) c- Measure and record the cardiac QRS complex d- Check current finger stick glucose 10- The nurse assesses an older adult who is newly admitted to a long term care facility. The client has dry, flaky skin and long thickened fingernails. The clients has a medical history of a stroke which resulted in left-sided paralysis and dysphagia. In planning care for the client, which task should the nurse delegate to the unlicensed personnel (UAP)? a- Soak and file fingernails b- Offer fluids frequently c- Monitor skin elasticity d- Ambulate in the hallway 11- A client is receiving lidocaine IV at 3 mg/minute. The pharmacy dispenses a 500 ml IV solution of normal saline (NS) with 2 grams of lidocaine. The nurse should regulate the infusion pump to deliver how many ml/hr? (Enter numeric value only. If rounding to the nearest whole number.) 45 12- The nurse is demonstrating wound care to a client following abdominal surgery. In what order should the nurse teach the technique? (Arrange from first action on top to last action on bottom) Remove old dressing using clean gloves. Discard gloves with old dressing Moisten sterile gauze with normal saline. Clean wound from least contaminated area to most contaminated area” Apply sterile gauze dressing to wound area Secure dressing with tape 13- The healthcare provider explains through an interpreter the risks and benefits of a scheduled surgical procedure to a non-English speaking female client. The client gives verbal consent and the healthcare provider leaves, instructing the nurse to witness the signature on the consent form. The client and interpreter then speaker together in the foreign language for an additional 2 minutes until the interpreter concludes, “She says it is OK.” What action should the nurse take next? a- Clarify the client‟s consent through the use of gesture and simple terms b- Have the interpreter co-sign the consent to validate client understanding c- Ask for full explanation from the interpreter of the witnessed discussion d- Have the client sign the consent and the nurse witness the signature 14- A client is admitted to a mental health unit after attempting suicide by taking a handful of medications. In developing a plan of care for this client, which goal has the highest priority? a- Signs a no-self-harm contract. b- Sleep at least 6 hours nightly. c- Attends group therapy every day d- Verbalizes a positive self-image. 15- After receiving report, the nurse can most safely plan to assess with client last? a- An older client with dark red drainage on a postoperative dressing, but no drainage in the Hemovacb- An adult client with no postoperative drainage in the Jackson- Pratt drain with the bulb compressed c- An older client with a distended abdomen and no drainage from the nasogastric tube d- An adult client with rectal tube draining clear pale red liquid drainage 16- The nurse is assigned to care for a client diagnosed with psoriasis. What behavior by the nurse addresses this client's psychosocial need for acceptance? a- Wearing gloves when interviewing the client b- Encouraging the client to join a support group c- Shaking the client's hand during an introduction. d- Allowing the client to ventilate feelings 17- A 41-week gestation primigravida woman is admitted to labor and delivery for induction of labor. What finding should the nurse report to the healthcare provider before initiating the infusion of oxytocin? a- Fetal heart tones located in upper right quadrant b- Biophysical profile results showing oligohydramnios c- Regular contractions occurring every 10 minutes d- Sterile vaginal exam reveling 3 cm dilatation 18- The nurse is preparing an older client for discharge following cataract extraction. Which instruction should be included in the discharge teaching? a- Do not read without direct lighting for 6 weeks b- Avoid straining at stool, bending, or lifting heavy objects. c- Irrigate conjunctiva with ophthalmic saline prior to installing antibiotic ointment. d- Limit exposure to sunlight during the first 2 weeks when the cornea is healing. 19- After learning that she has terminal pancreatic cancer, a female client becomes very angry and says to the nurse, “God has abandoned me. What did I do to deserve this?” Based on this response, the nurse decides to include which nursing problem in the client‟s plan of care? a- Ineffective coping b- Spiritual distress c- Acute pain d- Complicated grieving 20- Un infant is unresponsive and gasping for breath. Prior to starting CPR, which site should the nurse palpate for a pulse? C 21- A group of nurses implement a pilot study to evaluate a proposed evidence-based change to providing client care. Evaluation indicates successful outcomes and the nurses want to integrate the change throughout the facility. Which action should be taken? (Select all that apply) a- Invite data review by the quality improvement department b- Submit a sentinel event report to the research committee c- Propose clinical practice guidelines to the nursing committee d-Obtain informed consent from clients who will receive care e-Arrange inservice training through the educational department 22- The mother of a school age child calls the school to ask when her daughter can return to school after treatment for Pediculosis capitis. What is the nurse best response? (nits liendra) a- When the classroom epidemic subsides b- Two weeks after the last treatment c- As soon as the itching stops d- After the treatment kills all the live lice 23- A client in the intensive care unit is being mechanically ventilated, has an indwelling urinary catheter in place, and is exhibiting signs of restlessness. Which action should the nurse take first? a- Review the heart rhythm on cardiac monitors b- Check urinary catheter for obstruction c- Auscultated bilateral breath sounds d-Give PRN dose of lorazepam (Ativan) 24- What is the primary purpose for initiating nursing intervention that promote good nutrition, rest, and exercise, and stress reduction for clients diagnosed with an HIV infection? a- Prevent spread of infection to othersb- Improve function of the immune system c- Increase ability to carry out activities of daily living e- Promote a feeling of general well-being 25- When assessing a client with acute asthma, the nurse is most likely to obtain which finding? a- Pursed lip breathing and clubbing of fingers b- Fever and a high- pitched inspiratory stridor c- A short expiratory phase and hemoptysis d- Cough and musical breath sound on expiration 26- During the admission assessment, the nurse auscultates heart sounds for a client with no history of cardiovascular disease. Where should the nurse listen when assessing the client‟s point of maximal impulse (PMI) (Click the chosen location. To change, click on a new location) 27- Which medication should the nurse anticipate administering to a client who is diagnosed with myxedema coma? a- Intravenous administration of thyroid hormones b- Oral administration of hypnotic agents c- Intravenous bolus of hydrocortisone d- Subcutaneous administration of vitamin k 28- A client with multiple sclerosis is experiencing scotomas (blind spots), which are limiting peripheral vision. What intervention should the nurse include in this client's plan of care? a- Encourage the use of corrective lenses during the day b- Practice visual exercises that focus on a still object c- Alternate an eye patch from eye every 2 hoursd- Teach techniques for scanning the environment. 29- The nurse applies a blood pressure cuff around a client‟s left thigh. To measure the client‟s blood pressure, where should the diaphragm of the stethoscope be placed? (Mark the location on one of the images.) “On left thigh with arrow pointing to inner thigh” 30- Which intervention should the nurse include in the plan of care for a patient with tetanus? Open window shades to provide natural light a- Encourage coughing and deep breathing b- Minimize the amount of stimuli in the room c- Reposition from side to side every hour 31- The nurse is caring for a newborn who arrives in the nursery following a precipitous birth on the way to the hospital. A drug screen of the mother reveals the presence of cocaine metabolites. The infant has a heart rate of 175 beats/ minute, cries continuously, is irritable, and is hyperreactive to stimuli. Which intervention is most important for the nurse to include in this infant‟s plan of care? a- Initiate infant sepsis protocol b- Implements seizure precautions c- Refer to protective child services d- Formula feed every 3 hours 32- A client who is newly diagnosed with type 2 diabetes mellitus (DM) receives a prescription for metformin (Glucophage) 500 mg PO twice daily. What information should the nurse include in this client‟s teaching plan? (Select all that apply.) a- Take an additional dose for signs of hyperglycemia b- Recognize signs and symptoms of hypoglycemia. c- Report persist polyuria to the healthcare provider.d- Use sliding scale insulin for finger stick glucose elevation. e- Take Glucophage with the morning and evening meal. 33- A client with leukemia undergoes a bone marrow biopsy. The client‟s laboratory values indicate the client has thrombocytopenia. Based on this data, which nursing assessment is most important following the procedure? a- Observe aspiration site. b- Assess body temperature c- Monitor skin elasticity d- Measure urinary output 34- A client collapses while showering and is found discovered by the nurse while making rounds. The client is not breathing and does not have a palpable pulse. The nurse obtains the Automated External Defibrillator (AED). What action should the nurse implement next? a- Follow the prompts of the AED b- Apply the AED pads to the client’s chest c- Wipe the client’s chest dry d- Move the client from the bathroom 35- A female client with cancer tells the home care nurse that she has a good appetite but experiences nausea whenever she smells food cooking. What action should the nurse implement? a- Encourage family members to cook meals outdoors and bring the cooked food inside b- Advise the client to replace cooked foods with a variety of different nutritional supplements c- Assess the client‟s mucus membranes and report the findings to the healthcare provider d- Instruct the clients to take an antiemetic before every meal to prevent excessive vomiting. 36- A 13 years-old girl, diagnosed with diabetes mellitus Type 1 at the age of 9, is admitted to the hospital in diabetic ketoacidosis. Which occurrence is the most likely cause of the ketoacidosis? a- Ate an extra peanut butter sandwiches before gym class b- Incorrectly drew up and administered too much insulin c- Was not hungry, so she skipped eating lunch d- Has had a cold and ear infection for the past two days 37- At 1130, the nurse assumes care of an adult client with diabetes mellitus who was admitted with an infected foot ulcer. After reviewing the client‟s electronic health record, which priority nursing action should the nurse implement? Click on each chart tab for additional information. Please be sure to scroll to the bottom right corner of each tab to view all information contained in the client‟s medical record. a- Administer insulin per sliding scale b- Assess appearance of foot wound c- Obtain antibiotic peak and trough levels d- Initiate hourly urine output measurements 38- Following morning care, a client with C-5 spinal cord injury who is sitting in a wheelchair becomes flushed and complains of a headache. Which intervention should the nurse implement first? a- Relieve any kinks or obstruction in the client‟s Foley tubing b- Asses the client‟s blood pressures every 15 minutes c- Administer a prescribed PRN dose of hydralazine (Apresoline) e-Teach the client to recognize symptoms of dyreflexia 39- After a motor vehicle collision a client admitted to the medical unit with acute adrenal insufficiency (Addisonian crisis). Which prescription should the nurse implement? a- Determine serum glucose levels b- Withhold potassium additives to IV fluids c- Give IV corticosteroid replacement d- Prepare to initiate IV vasopressors 40- Which instruction is most important for the nurse to provide a client who receives a new prescription for risedronate sodium to treat osteoporosis a- Remain upright after taking the medication b- Increase intake of foods rich in calcium c- Begin a weight-bearing exercise plan e- Schedule a bone density test every year. 41- The unlicensed assistive personnel (UAP) reports that a client‟s blood pressure cannot be measured because the client has casts on both arms and is unable to be turned to the prone position for blood pressure measurement in the legs. What action should the nurse implement? a- Advise the UAP to document the last blood pressure obtained on the client graphic sheet b- Estimate the blood pressure by assessing the pulse volume of the client‟s radial pulses c- Demonstrate how to palpate the popliteal pulse with the client supine and the knee flexed d- Document why the blood pressure cannot be accurately measured at the present time42- The nurse identifies the presence of clear fluid on the surgical dressing of a client who just returned to the unit following lumbar spinal surgery. What action should the nurse implement immediately? a- Change the dressing using a compression bandage b- Test fluid on the dressing for glucose c- Document the findings in the electronic medical record d- Mark drainage area with a pen and continue monitor 43- Assessment by the home health nurse of an older client who lives alone indicates that client has chronic constipations. Daily medications include furosemide for hypertension and heart failure and laxatives. To manage the client‟s constipation, which suggestions should the nurse provide? (Select all that apply) a- Decrease laxative use to every other day, and use oil retention enemas as needed. b- Include oatmeal with stewed pruned for breakfast as often as possible. c- Increase fluid intake by keeping water glass next to recliner. d- Recommend seeking help with regular shopping and meal preparation. e- Report constipation to healthcare provider related to cardiac medication side effects. 44- A male client with diabetes mellitus takes NPH/ regular 70/30 insulin before meals and azithromycin PO daily, using medication he brought from home. When the nurse delivers his breakfast tray, the client tells the nurse that he took his insulin but forgot to take his daily dose of the azithromycin an hour before breakfast as instructed. What action should the nurse implement? a- Provide a PRN dose of an antacid to take with the azithromycin right after breakfast b- Offer to obtain a new breakfast tray in an hour so the client can take the azithromycin c- Instruct the client to eat his breakfast and take the azithromycin two hours after eating d- Tell the client to skip that day‟s dose and resume taking the azithromycin the next day 45- After administering a proton pump inhibitor (PPI), which action should the nurse take to evaluate the effectiveness of the medication? a- Ask the client about gastrointestinal pain b- Measure the client‟s fluid intake and output c- Monitor the client‟s serum electrolyte levels d- Auscultate for bowel sounds in all quadrants 46- Before leaving the room of a confused client, the nurse notes that a half bow knot was used to attach the client's wrist restraints to the movable portion of the client's bed frame. What action should the nurse take before leaving the room? a- Ensure that the knot can be quickly released. b- Tie the knot with a double turn or square knot. c- Move the ties so the restraints are secured to the side rails. d- Ensure that the restraints are snug against the client's wrist.47- An older male client arrives at the clinic complaining that his bladder always feels full. He complains of a weak urine flow frequent dribbling after voiding and increasing nocturia with difficulty initiating his urine stream action should the nurse implement? a- Palpate the client‟s suprapubic area for distention b- Advise the client to maintain a voiding diary for one week c- Instruct in effective techniques to cleanse the glans penis d- Obtain urine specimen for culture and sensitivity 48- A female client on the mental health unit frequently asks the nurse when she can be discharged. Then, becoming more anxious, she begins to pace the hallway. What intervention should the nurse implement first? a- Review the current treatment plan with the client b- Inform the healthcare provider about the client‟s behaviors c- Determine if the client has PRN medication for anxiety d- Explore the client‟s reasons for wanting to be discharged. 49- The nurse working on a mental health unit is prioritizing nursing care activities because of a staffing shortage. One practical nurse (PN) is on the unit with the nurse, and another RN is expected to arrive within two hours. Clients need to be awakened and morning medications need to be prepared. Which plan is best for the nurse to implement? a- Wake all the clients and instruct them go to dining area for medication administration b- Explain to the clients that it will be necessary to cooperate until another RN arrives c- Ask the PN to administer medications as clients are awakened so both nurses are available d- Allow the clients to sleep until a third staff person can assist with unit activities 50- A 5-week-old infant who developed projectile vomiting over the last two weeks is diagnosed with hypertrophic pyloric stenosis. Which intervention should the nurse plan to implement? a- Instruct the mother to give the child sugar water only b- Maintain intravenous fluid therapy per prescription c- Provide Pedialyte feedings via the nasogastric tube d- Offer the infant Pedialyte feedings every 2 hours. 51- A male client with hypertension, who received new antihypertensive prescriptions at his last visit returns to the clinic two weeks later to evaluate his blood pressure (BP). His BP is 158/106 mmHg and he admits that he has not been taking the prescribed medication because the drugs make him “feel bad”. In explaining the need for hypertension control, the nurse should stress that an elevated BP places the client at risk for which pathophysiological condition? a- Stroke secondary to hemorrhage b- Acute kidney injury due to glomerular damage c- Heart block due to myocardial damage d- Blindness secondary to cataracts52- . The nurse instructs an unlicensed assistive personnel (UAP) to turn an immobilized elderly client with an indwelling urinary catheter every two hours. What additional action should the nurse instruct the UAP to take each time the client is turned? a- Empty the urinary drainage bag b- Feed the client a snack c- Offer the client oral fluids e- Assess the breath sounds 53- A woman at 24-weeks gestation who has fever, body aches, and has been coughing for the las 5 days is sent to the hospital with admission prescriptions for H1N1 influenza. Which prescriptions has the highest priority? a- Ringers Lactate IV 125ml/8 hours b- Obtain specimens for cultures c- Assign private room d- Vital signs q4 hours 54- An older female client living in a low income apartment complex tells the home health nurse that she is concerned about her 81 – year old neighbor, a widow whose son recently assumed her financial affairs. Lately, her neighbor has become reclusive, but is occasionally seen walking outside wearing only robe and slippers. What response should the nurse offer? a- Explain that it is not unusual for older adults to suffer from dementia which often causes such behaviors b- Tell the client to talk to a healthcare provider before reporting suspicion of neglect to the authorities c- Provide the number for Adult Protective Services so the client can report any suspicion of elder abuse d- Encourage the client to avoid becoming involved the neighbor‟s problems, for one‟s own protection 55- A client with deep vein thrombosis (DVT) in the left leg is on a heparin protocol. Which intervention is most important for the nurse to include in this client's plan of care? a- Observe for bleeding side effects related to heparin therapy. b- Assess blood pressure and heart rate at least q4 hours c- Measure calf girths to evaluate edema in the affected leg d- Encourage mobilization to prevent pulmonary embolism 56- A nurse who took drugs from the unit for personal use was temporarily released from duty. After completion of mandatory counseling, the impaired nurse has asked nursing administration to allow return to work. When the nurse administrator approaches the charge nurse with the impaired nurse‟s request, what action is best for the charge nurse to take?a- Ask to meet with the impaired nurse‟s therapist before allowing the nurse back on the unit b- Meet with staff to assess their feelings about the impaired nurse‟s return to the unit c- Since treatment is completed, assign the nurse to routine RN responsibilities d- Allow the impaired nurse to return to work and monitor medication administration 57- A preschool teacher notifies the school nurse that child A has bitten child B on the arm. Child B‟s skin is broken, but is not bleeding. What action should the school nurse take first? a- Apply antibiotic cream to Child B‟s arm immediately b- Determine if Child A has a history of Hepatic C or HIV c- Determine the date of Child B‟s latest tetanus booster d- Wash Child B‟s arm thoroughly with soap and water 58- At a community health fair, a 50-year-old woman tells the nurse that she has an annual physical exam that includes a clinical breast exam and an annual mammogram. How should the nurse respond? a- Ask the woman if she also performs monthly breast self-exams. < b- Advise the woman that mammograms are only needed every two years at her age. c- Encourage the woman to explore her fears about breast cancer. d- Comment the woman for adhering to the recommended cancer detection guidelines. 59- (ESTA PREGUNTA TIENE DOS FORMAS DIFERENTES DE PREGUNTAR AQUI LES DEJO LAS DOS OPCIONES LA RESPUESTA ES LA MISMA) The practical nurse (PN) is assigned to work with three registered nurses (RN) who are caring for neurologically compromised clients. The client with which change in status is best to assign to the PN? The charge nurse is making assignments for one practical nurse (PN) and three registered nurses (RN) who are caring for neurologically compromised clients. which client with which change in status is best to assign to the PN? a- Diabetic ketoacidosis whose Glasgow Coma Scale score changed from 10 to 7 b- Subdural hematoma whose blood pressure changed from 150/80 mmHg to 170/60 mmHg c- Myxedema coma whose blood pressure changed from 80/50 mmHg to 70/40 mmHg d- Viral meningitis whose temperature changed from 101° F ( 38.3 C) to 102° F (38.9C). 60- Prior to surgery, written consent must be obtained. What is the nurse‟s legal responsibility with regard to obtaining written consent? a- Validate the clients understanding of the surgical procedure to be conducted b- Explain the surgical procedure to the client ask the client to sign the consent form c- Ask the client or a family member to sign the surgical consent form d- Determine that the surgical consent form has been signed and is included in the client‟s record61- The school nurse is screening students for spinal abnormalities and instructs each student to stand up and then touch the toes. Which finding indicates an a student should be referred for scoliosis evaluation? a- Inability to touch toes b- Asymmetry of the shoulders when standing upright c- Audible crepitus when bending d- An exaggerated upper thoracic convex curvature 62- The nurse is planning to assess a client's oxygen saturation to determine if additional oxygen is needed via nasal cannula. The client has a bilateral below-the-knee amputation and pedal pulses that are weak and threaty. What action should the nurse take? a- Document that an accurate oxygen saturation reading cannot be obtained b- Elevate to client's hands for five minutes prior to obtaining a reading from the finger c- Increase the oxygen based on the clients breathing patterns and lung sounds d- Place the oximeter clip on the ear lobe to obtain the oxygen saturation reading 63- The nurse provides sliding scale insulin administration instruction to an adult who was recently diagnose with diabetes mellitus. The client demonstrates and understanding on the instructions provided by performing the procedure in which order? (Arrange with the first on top and the last on the bottom.) Obtain blood glucose level Verify the insulin prescription Draw insulin into insulin syringe Cleanse the selected site 64- What is the primary focus of postoperative nursing care for the client with colon trauma? a- Monitoring for elevated coagulation studies b- Observation for and prevention of fistulas c- Monitoring for signs of hyponatremia d- Observation for and prevention of infection 65- While assessing a client four hours post-thoracentesis, the nurse is unable to auscultate breath sounds on the right side of the chest. What action should the nurse take first?a- Instruct the client to perform cough and deep breathing exercises b- Assess the client‟s vital signs and respiratory effort c- Administer oxygen per nasal canula per PNR protocol d- Document assessment findings in client‟s medical record 66- During discharge teaching, the nurse discusses the parameters for weight monitoring with a client who was recently diagnosed with heart failure (HF). Which information is most important for the client to acknowledge? a- Obtain at the same time every day b- Report weight gain of 2 pounds (0.9kg) in 24 hours c- Keep a daily weight record d- Limit intake of dietary salt 67- A client with osteoporosis related to long-term corticosteroid therapy receives a prescription for calcium carbonate. Which client‟s serum laboratory values requires intervention by the nurse? a- Total calcium 9 mg/dl (2.25 mmol/L SI) b- Creatinine 4 mg/dl (354 micromol/L SI) c- Phosphate 4 mg/dl (1.293 mmol/L SI) d-Fasting glucose 95 mg/dl (5.3 mmol/L SI) 68- A 3-year-old boy was successfully toilet trained prior to his admission to the hospital for injuries sustained from a fall. His parents are very concerned that the child has regressed in his toileting behaviors. Which information should the nurse provide to the parents? a- Children usually resume their toileting behaviors when they leave the hospital b- A retraining program will need to be initiated when the child returns home c- Diapering will be provided since hospitalization is stressful to preschoolers d- A potty chair should be brought from home so he can maintain his toileting skills 69- The nurse is caring for a group of a clients on a surgical unit. Which client should the nurse assess first? a- A client who is two days post knee surgery and who describes pain at “4” on a 1 to 10 scale b- A client who is one day post bowel resection with no bowel sounds c- A client who is 8 hours post appendectomy with urinary output of 480 ml d- A client who was admitted with severe abdominal pain and suddenly has no pain 70- A client who is mechanically ventilated is receiving continuous enteral feedings through a nasogastric feeding tube. To prevent aspiration, which intervention is most important for the nurse to implement?a- Verify the feeding tube position with a daily chest x-ray b- Maintain head of bed elevated while enteral feeding is infusing c- Check feeding tube placement with air bolus prior to use d- Aspirate stomach contents every 4 hours to assess residuals 71- After years of struggling with weight management, a middle-age man is evaluated for gastroplasty. He has experienced difficulty with managing his diabetes mellitus and hypertension, but he is approved for surgery. Which intervention is most important for the nurse to include in this client‟s plan of care? a- Monitor for urinary incontinence b- Apply sequential compression stockings c- Provide a wide variety of meal choices d- Observe for signs of depressions 72- A client with diabetes mellitus tells the nurse that she uses cranberry juice to help prevent urinary tract infection. What instruction should the nurse provide? a- Be sure to drink sugar-free cranberry juice b- Drinking cranberry juice does not prevent infection c- Cranberries may increase the effect of insulin d- Excess cranberry juice can be constipating 73- After receiving report on an inpatient acute care unit, which client should the nurse assess first? a- The client with bowel obstruction due to a volvulus who is experiencing abdominal rigidity b- The client who had surgery yesterday and is experiencing a paralytic ileus with absent bowel sounds c- The client with an obstruction of the large intestine who is experiencing abdominal distention d- The client with a small bowel obstruction who has a nasogastric tube that is draining greenish fluid 74- An adolescent male client is admitted to the hospital. Based on Erikson‟s theory of psychosocial development, which nursing intervention best assists this adolescent‟s adjustment to his hospital stay? a- Invite him to participate in the evening group activity b- Schedule frequent private phone calls to his parents c- Provide access to a variety of video games in his room d- Encourage him to learn his way around the hospital 75- A client with skin grafts covering full-thickness burns on both arms and legs is scheduled for a dressing change. The client is nervous and requests that the dressing change be skipped this time. What action is most important for the nurse to take? a- Explain the importance of regular dressing change b- Administer an anti-anxiety medication c- Proceed with the scheduled dressing change d- Encourage the client to express any anxieties 76- A continuous infusion of nitroglycerin is prescribed for an adult male admitted with an acute myocardial infarction. The client is experiencing active chest pain that he describes as 8 out of 10. Which intervention is most important for the nurse to implement? a- Administer infusion via an infusion pump b- Obtain current serum potassium level c- Continuously monitor blood pressure d- Teach guided imagery to decreased pain 77- When caring for a client with diabetes insipidus (DI), it is most important for the nurse to include frequent assessment for which conditions in the client‟s plan of care? a- Dry mucous membranes, hypotension b- Decreased appetite, headache c- Nausea and vomiting, muscle weakness d- Elevated blood pressure, petechiae 78- The nurse administers an oral antiviral to a client with shingles. Which finding is most important for the nurse report to the healthcare provider? a- Decreased white blood cell count b- Pruritus and muscle aches c- Elevated liver function tests e- Vomiting and diarrhea 79- A child with heart failure (HF) is taking digitalis. Which signs indicates to the nurse that the child may be experiencing digitalis toxicity? a- Tachycarcia b- Dyspnea c- Vomiting d- Muscle cramps 80- A client is admitted with hepatitis A (HAV) and dehydration. Subjective symptoms include anorexia, fatigue, and malaise. What additional assessment should the nurse expect to find during the preicteric phase? a- RUQ abdominal pain b- Clay – colored stool c- Icteric sclera d- Pruritis81- An older male was recently admitted to the rehabilitation unit with unilateral neglect syndrome as the result of a cerebrovascular accident (CVA). Which action should the nurse include in the plan of care? a- Provide additional light in the room to promote sensory stimulation b- Teach the client to turn his head from side to side for visual scanning c- Place a clock and calendar in the room to improve orientation d- Use hand and arm gestures to improve communication and comprehension 82- The nurse is caring for a client with a tracheostomy. Which action should the nurse perform when suctioning the tracheostomy tube? a- Increase wall suction with removal of the suction catheter b- Place sterile saline 10 ml in the tracheostomy tube, then suction as continuous suction is applied c- Suction the client‟s oropharynx before tracheal suctioning d- Insert the suction catheter into the trachea, and apply intermittent suction with removal catheter 83- The urinary drainage of a client with a continuous bladder irrigation is becoming increasingly red. Which intervention should the nurse implement? a- Increase the irrigation rate b- Lower the head of the bed c- Milk the catheter tubing d- Evaluate for fluid overload 84- The nurse is preparing to administer a suspension ampicillin labeled, 250mg/5ml, to a 12-year old child with impetigo. The prescription is for 500 mg QID. How many ml should the child receive per day? (Enter numeric value only) 40 85- A client with a severe prostatic infection that caused a blocked urethra is 3 days post-surgical urinary diversion. The healthcare provider directs the nurse to remove the suprapubic catheter to allow the client to void normally. Which intervention should the nurse implement first? a- Cleanse the site around the catheter b- Use a 20 ml syringe to deflate balloon c- Clamp catheter until a client voids naturally d- Empty urine from urinary drainage bag 86- The nurse receives report on four clients who are complaining of increased pain. Which client requires immediate by the nurse ? a- Burning pain due to a Morton‟s neuromab- Sharp pain related to a crushed femur c- Paresthesia of fingers due to carpal tunnel syndrome d- Stinging pain related to Plantar fasciitis 87- A client morning assessment includes bounding peripheral pulses, weight gain of 2 pounds, pitting ankle edema, and moist crackles bilaterally. Which intervention is most important for nurse to include the in this client‟s plan of care? a- Restrict daily fluid intake to 1500 ml b- Weight client every morning c- Maintain accurate intake and output d- Administer prescribed diuretic 88- A female client who has a borderline personality disorder is being discharged today. When the nurse makes morning rounds, the clients begins the interaction by complaining about the aloofness of the night shift nurse and expresses joy to see that, „‟ My favorite nurse is on duty now” Which response is best for the nurse to provide to this client‟s dichotomous tendency? a- I am glad you like me. Which nurse was acting aloof to you? b- Tomorrow I will talk to that nurse about how you were treated last night c- What did the night nurse that makes you think she is aloof? d- I am happy that you are getting better and will be able to go home 89- The nurse is calculating the one-minute Apgar score for a newborn male infant, and determines that his heart rate is 150 beats/minute, he has a vigorous cry, his muscle tone is good with total flexion, he has quick reflex irritability, and his color is dusky and cyanotic. What Apgar score should the nurse assign to the infant? (Enter the numeric value only.) 8 90- The nurse is assigned to care for four surgical clients. After receiving report, which client should the nurse see first? a- An older client who is receiving packed RBCs on the third day postoperatively for colon resection b- An older client with continuous bladder irrigation who is 2 day postoperatively for bladder surgery c- An adult one day postoperatively laparoscopic cholecystectomy requesting pain medication d- An adult who is in Buck‟s traction, and scheduled for hip arthroplasty within the next 12 hours. 91- While changing a client‟s chest tube dressing, the nurse notes a crackling sensation when gentle pressure is applied to the skin at the insertion site. What is the best action for the nurse to take? a-Apply a pressure dressing around the chest tube insertion site.b-Assess the client for allergies to topical cleaning agents. c-Measure the area of swelling and crackling. d-Administer an oral antihistamine per PRN protocol. 92- When teaching a group of school-age children how to reduce the risk of Lyme disease which instruction should the camp nurse include? a- Wash hands frequently b- Avoid drinking lake water c- Wear long sleeves and pants e- Do not share personal products 93- In preparing a nursing care plan for a client admitted with a diagnosis of Guillain-Barre syndrome, which nursing problem has the highest priority? a- Ineffective coping related to uncertainly of disease progression b- Imbalanced nutrition: less than body requirements related to impaired swallowing reflex c- Ineffective breathing pattern related to ascending paralysis. d- Impaired physical mobility related to asymmetrical descending paralysis 94- A newly hired unlicensed assistive personnel (UAP) is assigned to a home healthcare team along with two experienced UAPs. Which intervention should the home health nurse implement to ensure adequate care for all clients? a- Ask the most experienced UAP on the team to partner with the newly hired UAP b- Evaluate the newly hired UAP‟s level of competency by observing the UAP deliver care c- Review the UAP‟s skills checklist and experience with the person who hired the UAP d- Assign the newly hired UAP to clients who require the least complex level of care 95- What Nursing intervention is particularly indicated for the second stage of labor? a- Providing pain medication to increase the client‟s tolerance of labor b- Assessing the fetal heart rate and pattern for signs of fetal distress c- Monitoring effects of oxytocin administration to help achieve cervical dilation d- Assisting the client to push effectively so that the expulsion of the fetus can be achieved. 96- An adult male with a 6 cm thoracic aneurysm is being prepared for surgery. The nurse reports to the healthcare provider that the client‟s blood pressure is 220/112 mmHg, so an antihypertensive agent is added to the client‟s IV infusion. Which finding warrants immediate intervention by the nurse? a- Reports a tearing, sharp pain between his shoulder blades b- Blood pressure reading of 200/100 mmHg 15 minutes later c- Rose colored urine draining from the urinary catheter d- Sinus tachycardia with frequent premature ventricular beats (PVC)97- When entering a client‟s room to administer an 0900 IV antibiotic, the nurse finds that the client is engaged in sexual activity with a visitor. Which actions should the nurse implement? a- Ignore the behavior and hang the IV antibiotic b- tell the client to stop the inappropriate behavior c- Leave the room and close the door quietly d-Complete an unusual occurrence report 98- A client is admitted with the diagnosis of Wernicke‟s syndrome. What assessment finding should the nurse use in planning the client‟s care? a- Right lower abdominal pain b- Confusion c- Depression d- Peripheral neuropathy 99- The nurse is developing a teaching plan for a client with acute gastritis caused by drinking contaminated water. The nurse should emphasize the need to report the onset of which problem? a- Low grade fever b- Bruising of the skin c- Abdominal cramping d- Bloody emesis 100- A client with complaints of shortness of breath and abdominal pain 1 week after bariatric surgery is admitted for follow-up evaluation. Which assessment finding warrants immediate intervention by the nurse? a- Rectal temperature of 101F b- Complaints of left shoulder pain c- Blood pressure of 88/50 mmHg d- Sustained sinus tachycardia 101- A client with prescription for “do not resuscitate” (DNR) begins to manifest signs of impending death. After notifying the family of the client‟s status, what priority action should the nurse implement? a- The client‟s need for pain medication should be determined b- The impeding signs of death should be documented c- The nurse manager should be update on the client‟s status d- The client‟s status should be conveyed to the chaplain. 102- An adult male is admitted to the intensive care unit because he experienced a sudden onset of sharp chest pain and shortness of breath earlier today. Following an emergent pulmonary angiogram, the client is diagnosed with a pulmonary embolism. Which intervention is most important for the nurse to include in this client‟s plan of care? a- Monitor signs of increased bleeding b- Instruct on the use of incentive spirometry c- Observe for confusion and restlessnessd- Administer intravenous opioids for severe pain 103- A female client presents in the Emergency Department and tells the nurse that she was raped last night. Which question is most important for the nurse to ask? a-Does she knows the person who raped her? b-Has she taken a bath since the raped occurred? c-Is the place where she lived a safe place? d-Did she report the rape to the police Department? 104- A client is receiving lactulose (Portalac) for signs of hepatic encephalopathy. To evaluate the client's therapeutic response to this medication, which assessment should the nurse obtain? a- Level of consciousness. b- Percussion of abdomen c- Serum electrolytes d- Blood glucose. 105- A male client with schizophrenia tells the nurse that the hospital has installed cameras that watch him and listening devices that record what everyone says. Which nursing problem should the nurse document for this client? a- Noncompliance with medication related to thought broadcasting b- Situational self esteem disturbance secondary to schizophrenia c- Disturbed sensory perception related to auditory hallucinations d- Impaired environmental interpretation related to paranoid delusions 106- A nurse is administering diazepam, a benzodiazepine, 10 mg IV push PRN, as prescribed to a client with alcohol withdrawal symptoms. Which actions should the nurse implement when administering the medication? (Select all that apply) a- Protect medication from exposure to light b- Monitor for changes in level of consciousness c- Observe for onset of generalized bruising or bleeding d- Perform ongoing assessment of respiratory status e- Administer slowly over at least two minutes 107- An older male client is admitted to the mental health unit with a sudden onset of global disorientation and is continuously conversing with his mother, who died 50 years ago. The nurse reviews the multiple prescriptions the client is currently taking and assesses his urine specimen, which is cloudy, dark yellow, and has a foul odor. These findings suggest that this client is experiencing which condition? a- Psychotic episode b- Dementia c- Delirium d- Depression 108- While changing the pressure ulcer dressing of a client who is immobile, the nurse notes that the boundary edges of the wound have increased. Before reporting this finding to the healthcare provider, the nurse should review which of the client‟s serum laboratory values?a- Potassium b- Platelets c- Creatinine d- Albumin 109- A client is admitted for an exacerbation of heart failure (HF) and is being treated with diuretics for fluid volume excess. In planning nursing care, which interventions should the nurse include? (Select all apply) a- Encourage oral fluid intake of 3,000 ml/day b- Observe for evidence of hypokalemia c- Teach the client how to restrict dietary sodium d- Monitor PTT, PT, and IRN, lab values e- Weight the client daily, in the morning 110- A 17-year –old adolescent is brought to the emergency department by both parents because the adolescent has been coughing and running a fever with flu-like symptoms for the past 24 hours. Which intervention should the nurse implement first? a- Obtain a chest X-ray per protocol. b- Place a mask on the client‟s face. c- Assess the client‟s temperature. d-Determine the client‟s blood pressure 111- The nurse provides discharge teaching to a client who was recently diagnosed with diabetes mellitus (DM). After receiving the instructions, the client expresses understanding about when, how, and why to take his prescribed medications at home. Which intervention is most important for the nurse to implement a- Review the purpose of medications prescribed for the client to take home with him b- Provide the client with a printed list of medications and a schedule for administration. c- Send a list of medications taken while hospitalized to the client‟s healthcare provider d- Offer to consult with the pharmacist about resources for reduced price medications 112- The nurse is conducting intake interviews of children at a city clinic. Which child is most susceptible to contracting lead poisoning? a- An adolescent who works part time in a pain factory b- A 2- year- old who plays on aging outdoor playground equipment c- A 10 – year – old who has Type 1 diabetes mellitus d-An 8 – year old who lives in a housing project 113- An IV infusing in a client‟s left forearm becomes infiltrated. After removing the IV, which sites should the nurse select as possible site to insert another IV catheter? (Select all that apply) a- Right hand b- Right forearm c- Left hand d- Right subclavian e- Left subclavian114- An IV antibiotic is prescribed for a client with a postoperative infection. The medication is to be administered in 4 divided doses. What schedule is best for administering this prescription? a- 1000, 1600, 2200, 0400. b- Give in equally divided doses during waking hours c- Administer with meals and a bedtime snack d- 0800, 1200, 1600, 2000 115- A college student brings a dorm roommate to the campus clinic because the roommate has been talking to someone who is not present. The client tells the nurse that her voices are saying, “Kill, Kill” What question should the nurse ask the client next? a- When did these voices begin? b- Are you planning to obey the voices? c- Have you taken any hallucinogens? d- Do you believe the voices are real? 116- A client has had several episodes of clear, watery diarrhea that starter yesterday. What action should the nurse implement? a- Administer a prescribed PRN antiemetic b- Assess the client for the presence of hemorrhoids c- Check the client‟s hemoglobin level d- Review the client‟s current list of medications 117- The nurse notes that a depressed female client has been more withdrawn and noncommunicative during the past two weeks. Which intervention is most important to include in the updated plan of care for this client? a- Encourage the client‟s family to visit more often b- Schedule a daily conference with the social worker c- Encourage the client to participate in group activities d- Engage the client in a non-threatening conversation. 118- A postpartum client who is bottle feeding develops breast engorgement. What is the best recommendation for the nurse to provide this client? a- Take a prescribed analgesic and exposed breasts to air b- Place warm packs on both of the breasts c- Avoid stimulation of the breasts and wear tight bra d- Express a small amount of breast milk by hand 119- A nurse assesses a client whose hand begins to spasm when the blood pressure cuff is inflated. The client complains of paresthesia in the fingers and toes. Which serum laboratory finding should the nurse expect to find?a- Elevated serum calcium b- Low serum magnesium c- Low serum calcium d- Elevated serum potassium 120- The parents of a 6-year-old child recently diagnosed with Duchenne muscular dystrophy tell the nurse that their child wants to continue attending swimming classes. How should the nurse respond? a- Encourage the parents to allow the child to continue attending swimming lesson with supervision b- Suggest that the child be encouraged to participate in a team sport to encourage socialization c- Explain that their child is too young to understand the risks associated with swimming d- Provide a list of alternative activities that are less likely to cause the child experience fatigue 121- An older male resident of a long-term care facility has been scratching his legs for the past 2 days. Which intervention should the nurse implement? a- Explain the importance of bathing or showering daily b- Encourage fluid intake of a least 2,000 ml daily c- Keep the legs covered as much as possible d- Apply emollient to affect area at least twice daily. 122- In performing the admission assessment for a client experiencing complications of long term Parkinson‟s disease, which question by the nurse provides the best information about disease progression? a- Have you experienced any stiffness in your neck or shoulder?” b- “Do you notice any jerky type movements of your arms?” c- “Have you ever been frozen to a spot and unable to move?” < d- “Do you have any problems with your hands shaking?” 123- An adult suffered burns to face and chest resulting from a grease fire. On admission, the client was intubated and a 2 liter bolus of normal saline was administered IV. Currently the normal saline is infusing at 250ml/hour. The client‟s heart rate is 120 beats/minute, blood pressure is 90/50 mmHg, respirations are 12 breaths/minute over the ventilated 12 breaths for a total of 24 breaths/minute, and the central venous pressure (CVP)is 4 mm H2O. Which intervention should the nurse implement? a- Increase the oxygen delivered by the ventilator b- Infuse an additional bolus of normal saline c- Lower head of the bed to a recumbent position d- Bring a tracheotomy tray to the bedside124- The nurse is caring for a client who has silvery scaling plaques bilaterally on elbows, forearms, and palms. When scratched, the skin bleeds over these plaques. What is most important to include in this client‟s plan of care? a- Interventions to decrease emotional stress b- Precaution about avoiding sunlight c- Strategies to increase omega-3 fatty acids in the diet d- Instruct about applying an antifungal ointment 125- After successful resuscitation, a client is given propranolol and transferred to Intensive Coronary Care Unit (ICCU). On admission, magnesium sulfate 4 grams IV in 250 ml D5 W at one gram/hour. Which assessment findings require immediate intervention by the nurse? a- Dark amber urine draining per indwelling catheter with 40 ml per hour b- Serum calcium of 9.0 mg/dl (2.2 mmol/L SI) and magnesium of 1.8 md/dl or Eq/L (0.74 mmol/L SI) c- Sinus rhythm at 72 beats/minute and peripheral blood pressure of 99/62 d- Respiratory rate of 10 breaths per minute and pulse oximetry of 90%. 126- A small, round raised are appears under the client‟s skin as the nurse administers an intradermal medication. What actions should the nurse take? a- Apply a col pack to the area for twenty minutes b- Elevate the area and apply light pressure over the site c- Document the site where the medication was given d- Notify the healthcare provider of the allergic response 127- A client who has been newly diagnosed with abnormally high production of antidiuretic hormone reports muscle cramps and twitching. The client also reports a headache and lethargy. These concerns are most likely related to which electrolyte imbalance? a- Hypermagnesium b- Hyponatremia c- Hypokalemia d- Hypercalcemia 128- The nurse asks the parent to stay during the examination of a male toddler‟s genital area. Which intervention should the nurse implement? a- Examine the genitalia as the last part of the total exam b- Use soothing statements to facilitate cooperation c- Allow child to keep underpants on to examine the genitalia d- Work slowly and methodically so not to stress the child 129- A woman at 12-week gestation comes to the clinic for her first prenatal visit. After completing a health history, the nurse should discuss which topic about pregnancy at this initial visit? a- Complications associated with childbirth b- Concerns about parenting c- Cultural practices related to childbearing d- Knowledge about labor and delivery 130- When preparing to transfer a young adult who has a spinal cord injury (SCI) to a rehabilitation center, which information about the client is most important for the nurse to provide to the receiving agency? a- The degree of family support available to the client b- The client‟s current ability to participate in daily care c- The client‟s remaining length of insurance coverage d- A description of the client‟s attitude about the transfer 131- A client was admitted to the cardiac observation unit 2 hours ago complaining of chest pain. On admission, the client‟s EKG showed bradycardia, ST depression, but no ventricular ectopy. The client suddenly reports a sharp increase in pain, telling the nurse, “I feel like an elephant just stepped on my chest” The EKG now shows Q waves and ST segment elevations in the anterior leads. What intervention should the nurse perform? a-Increase the peripheral IV flow rate to 175 ml/hr to prevent hypotension and shock b-Administer prescribed morphine sulfate IV and provide oxygen at 2 L/min per nasal cannula. c- Obtain a stat 12 lead EKG and perform a venipuncture to check cardiac enzymes levels. d-Notify the healthcare provider of the client‟s increase chest pain a call for the defibrillator crash cart. 132- A client newly diagnosed with diabetes mellitus suddenly becomes confused and weak. Which interventions should the nurse implement? (Select all that apply) a- Give the client 4 ounces of orange juice b- Obtain blood pressure and pulse rate c- Provide the client with ½ cup diet carbonated soda d- Administer a PRN dose of regular insulin e- Check the client‟s current finger stick blood glucose 133- The nurse assesses a client who had bilateral total knee replacements (TKR) four hours ago the nurse notes that the dressing on the client‟s right knee is saturated with serosanguineous drainage. What action should the nurse implement? a- Determine if the wound drainage device is functioning correctly b- Confirm that the continuous passive motion device is intact c- Withhold next scheduled dose of low molecular weight heparin d- Monitor the clients current white blood cell count (WBC)134- An older adult with known cognitive impairment residing in a long- term care facility suddenly becomes disoriented and confused. There are no signs of extremity weakness or other neurological changes. Which intervention should the nurse implement? a- Obtain 12 lead electrocardiogram b- Assess the urine for cloudiness c- Perform stroke assessment d- Auscultate for bowel sounds 135- A client with multiple sclerosis (MS) has decreased motor function after taking a hot bath (Uhthoff‟s sign). Which pathophysiological mechanism supports this response? a- Arterial Constriction b- Temporary vasodilation c- Poor temperature control d-Severe dehydration. 136- The husband of an older woman, diagnosed with pernicious anemia, calls the clinic to report that his wife still has memory loss and some confusion since she received the first dose of nasal cyanocobalamin two days ago. He tells the nurse that he is worried that she might be getting Alzheimer‟s disease. What action should the nurse take? a-Explain that memory loss and confusion are common with vitamin B12 deficiency b- Encourage the husband to bring the client to the clinic for a complete blood count c- Determine if the client is taking iron and folic acid supplements d-Ask if the client is experiencing any change in bowel habits 137- A client is receiving enoxaparin 30 mg SUBQ BID. In assessing for adverse effects of the medication, which serum laboratory value is most important for the nurse to monitor? a- White blood cell count b- Platelet count c- Glucose d- Calcium 138- A female client with severe renal impairment is receiving enoxaparin (lovenox) 30 mg SUBQ BID. Which laboratory value due to enoxaparin should the nurse report to the healthcare provider? a- creatinine clearance 25 mL/ minute b- calcium 9 mg/dl c- hemoglobin 12 grams/dl d- partial thromboplastin time (PTT) 30 seconds 139- After receiving the morning report at 0700 on a postpartum unit, which client should the nurse assess first? The client who is a- Complaining of IV site pain whose IV antibiotic is due at 0800 b- 23-hours postpartum and complaining of episiotomy pain. c- A third day post- cesarean requesting assistance to the bathroom d- Requesting assistance with breastfeeding her one-day old infant 140- Which laboratory results should the nurse closely monitor in a client who has end – stage renal disease (ESRD) a- Serum Potassium, calcium, and phosphorus b- Erythrocytes, hemoglobin, and hematocrit c- Blood pressure, heart rate and temperature d-Leukocytes, neutrophils and thyroxine 141- Which assessment finding is most important when planning to provide a complete bed bath to a bedfast client? a- 2+ pitting edema of the feet b- Right-side paralysis c- Orthopnea d- Pallor 142- The nurse observes an unlicensed assistive personnel (UAP) applying an alcohol-based hand rub while leaving a client‟s room after taking vital signs. What action should the nurse take? a- Remind the UAP to continue rubbing the hands together until they dry b- Instruct the UAP to return to the client‟s room to perform handwashing c- Advise the UAP to wear gloves when obtaining vital signs for all clients d- Supervise the UAP in the next client‟s room to evaluate hand hygiene 143- Which client problem has the higher priority for the child sickle cell anemia who has a temperature of 101 F (38.3)? a- Infection related to low platelet count b- Activity intolerance related to anemia c- Fluid volume deficit related to temperature elevation d- Altered urinary elimination related to renal damage 144- A client with chronic kidney disease receives a prescriptions for darbepoetin alfa 40 mcg subcutaneous every 7 days. The darbepoetin alfa vial is labeled, “60 mcg/ml” How many ml should the nurse administer ? (Enter numeric value only. If rounding is required, round to the nearest tenth) 0.7145- A client with metastatic cancer who was taking hydromorphone (Dilaudid) PO at home is now receiving the medication IV while in the hospital. To evaluate if the client is receiving an equianalgesic dose of the Dilaudid, what assessment should the nurse complete? a- Pain scale b- Level of consciousness c- Respiratory rate d- Blood pressure 146- When planning care for a client with acute pancreatitis, which nursing intervention has the highest priority? a- Withhold food and fluid intake. b- Initiate IV fluid replacement. c- Administer antiemetic as needed. d- Evaluate intake and output ratio. 147- What instruction should the nurse provide to a client who is preparing to have a cystoscopy ? a- Avoid strenuous activity and sports for a least 2 weeks b- Report any allergies to shellfish or iodine c- Lay prone for 24 hours after the procedure d- Report any painful urination, blood urine, or fever 148- While assessing a client‟s chest tube (CT), the nurse discovers bubbling in the water seal chamber of the chest tube collection device. The client‟s vital signs are: blood pressure of 80/40 mmHg, heart rate 120 beats/minutes, respiratory rate 32 breaths/minutes, oxygen saturation 88%. Which interventions should the nurse implement? a- Provide supplemental oxygen b- Auscultate bilateral lung fields c- Administer a nebulizer treatment d- Reinforce occlusive CT dressing e- Give PRN dose of pain medication 149- Nurses working on a surgical unit are concerned about the physicians treatment of clients during invasive procedures, such as dressing changes and insertion of IV lines. Clients are often crying during the procedures, and the physician is usually unconcerned or annoyed by the client‟s response. To resolve this problem, what actions should the nurses take? (Arrange from the first action on the top of the list on the bottom) RESPONDIDA Talk to the physician as a group in a non-confrontational manner. Document concerns and report them to the charge nurseSubmit a written report to the director of nursing. Contact the hospital‟s chief of medical services. File a formal complaint with the state medical board. 150- While transferring a client with a chest tube from the bed to a stretcher, the chest becomes disconnected from the water-seal drainage container. The nurse immediately immerses the end of the tube in a container of sterile water. What action should the nurse implement next? a- Begin manual ventilation while returning the client to the bed b- Clamp the chest tube and maintain its distal end in the water c- Prepare a new water-seal system and reattach the chest tube d- Apply pressure to the chest tube site using a petroleum gauze. 151- After diagnosis and initial treatment of a 3-year-old with Cystic fibrosis, the nurse provides home care instructions to the mother, which statement by the child's mother indicates that she understands home care treatment to promote pulmonary functions? a- Chest physiotherapy should be performed twice a day before a meal b- Energy should be conserved by scheduling minimally strenuous c- Administer a cough suppressant every 8 hour d- Maintain supplemental oxygen at 4 to 6 L/minute 152- The nurse is assessing a first day postpartum client. Which finding is most indicative of a postpartum infection? a- Oral temperature of 100.2 F (37.9 C) b- Blood pressure of 122/74 mmHg c- Moderate amount of foul-smelling lochia. d- White blood count of 19,000mm3 (19x10/L SI units) 153- The nurse is performing a peritoneal dialysis exchange on a client with chronic kidney disease (CKD). Which assessment finding should the nurse report to the healthcare provider? a- The appearance of the returning dialysate fluid is cloudy b- The client complains of a slight shortness of breath during installation c- The amount of the returning dialysate fluid is greater than the amount instilled d- The client complains of abdominal fullness and cramping during instillation154- A client with type 2 diabetes mellitus is admitted for frequent hyperglycemic episodes and a glycosylated hemoglobin (HbA1c) of 10%. Insulin glargine 10 units subcutaneously once a day at bedtime and a sliding scale with insulin aspart q6h are prescribed. What action should the nurse include in this client‟s plan of care? a- Fingerstick glucose assessment q6h with meals b- Mix bedtime dose of insulin glargine with insulin aspart sliding scale dose c- Review with the client proper foot care and prevention of injury d- Do not contaminate the insulin aspart so that it is available for iv use e- Coordinate carbohydrate controlled meals at consistent times and intervals f- Teach subcutaneous injection technique, site rotation and insulin management 155- At 1615, prior to ambulating a postoperative client for the first time, the nurse reviews the client‟s medical record. Based on date contained in the record, what action should the nurse take before assisting the client with ambulation: a- Remove sequential compression devices. b- Apply PRN oxygen per nasal cannula. c- Administer a PRN dose of an antipyretic. d- Reinforce the surgical wound dressing. 156- The nurse administers an antibiotic to a client with respiratory tract infection. To evaluate the medication‟s effectiveness, which laboratory values should the nurse monitor? Select all that apply a- White blood cell (WBC) count b- Sputum culture and sensitivity c- droplet precautions e- protective environment 157- A male client with angina pectoris is being discharged from the hospital. What instructions should the nurse plan to include to the discharge teaching? a- Engage in physical exercise immediately after eating to help decrease cholesterol levels. b- Walk briskly in cold weather to increase cardiac output. c- Keep nitroglycerin in a light-colored plastic bottle and readily available. d- Avoid all isometric exercises, but walk regularly.158- The nurse offers diet teaching to a female college student who was diagnosed with irondeficiency anemia following her voluntary adoption of a lacto-vegetarian diet. What nutrients should the nurse suggest this client eat to best meet her nutritional needs while allowing her to adhere to a lacto-vegetarian diet? a- Combine several legumes and grains such as beans and rice form complete proteins b- Drink whole milk instead of skim milk to enhance the body´s production of amino acids c- Increase amount of dark yellow vegetables such carrots to fortify iron stores d- Take vitamin K 10mg PO daily to daily to enhance production of red blood cells 159- The public nurse health received funding to initiate primary prevention program in the community. Which program the best fits the nurse‟s proposal? a- Case management and screening for clients with HIV. b- Regional relocation center for earthquake victims c- Vitamin supplements for high-risk pregnant women. d- Lead screening for children in low-income housing. 160- A client is admitted to the labor and delivery unit early labor and the nurse assesses the status of her contractions. The frequency of contractions is most accurately evaluated by counting the minutes and seconds in which manner? a- From the peak of one contraction to the peak of the next contraction b- From the beginning of one contraction to the end that contraction c- From the beginning of one contraction to the beginning of the next contraction d- From the end one contraction to the beginning of the next contraction 161- The nurse is preparing to send a client cardiac catheterization lab for an angioplasty. Which client report is most important for the nurse to explore further prior to start of the procedure? a- Verbalizes a fear of being in a confined space. b- Drank a glass of water in the past 2 hours. c- Reports left chest wall pain prior to admission. d- Experiences facial swelling after eating crab 162- A client who is admitted to the intensive care unit with syndrome of inappropriate antidiuretic hormone (SIADH) has developed osmotic demyelination. Which intervention should the nurse implement first? a- Patch one eye. b- Reorient often. c- Range of motion. d- Evaluate swallow163- During the administration of albuterol per nebulizer, the client complains of shakiness. The client´s vital signs are heart rate 120 beats/minutes, respirations 20 breaths/minute, blood pressure 140/80. What action should the nurse take? a- Administer anxiolytic b- Obtain 12 lead electrocardiogram c- Stop the albuterol administration and restart in 30 minutes d- Educate client about the side effects of albuterol 164- When attempting to establish risk reduction strategies in a community, the nurse notes that regional studies indicate a high number of persons with growth stunting and irreversible mental deficiencies caused by hypothyroidism (cretinism). The nurse should seek funding to implement which screening measure? a- T3 levels in school-aged children b- T4 levels in newborn c- TSH level in women over 45 d- Iodine levels in all persons over 60 165- While teaching a young male adult to use an inhaler for his newly diagnosed asthma, the client stares into the distance and appears to be concentrating on something other than the lesson the nurse is presenting. What action should the nurse take? a- Remind the client that a rescue inhaler might save his life b- Leave the client alone so that he can grieve his illness c- Ask the client what he is thinking about at his time. d- Gently touch the client then continue with teaching. 166- What should the nurse conduct an Allen´s test? a- When pulmonary artery pressures are obtained b- To assess for presence of a deep vein thrombus in the leg c- Just before arterial blood gasses are drawn peripherally d- Prior to attempting a cardiac output calculation 167- A client with acute renal failure (ARF) is admitted for uncontrolled type 1 diabetes Mellitus and hyperkalemia. The nurse administers an IV dose of regular insulin per sliding scale. Which intervention is the most important for the nurse to include in this client‟s plan of care? a- Monitor the client‟s cardiac activity via telemetry. b- Maintain venous access with an infusion of normal saline. c- Assess glucose via finger stick q4 to 6 hours. d- Evaluate hourly urine output for return of normal renal function. 168- An adult male was diagnosed with stage IV lung cancer three weeks ago. His wife approaches the nurse and asks how she will know that her husband's death is imminent because their two adult children want to be there when he dies. What is the best response by the nurse?a- Explain that the client will start to lose consciousness and his body system will slow down b- Reassure the spouse that the healthcare provider will let her know when to call the children c- Offer to discuss the client‟s health status with each of the adult children d- Gather information regarding how long it will take for the children to arrive 169- An S3 heart sound is auscultated in a client in her third trimester of pregnancy. What intervention should the nurse take? a- Notify the healthcare provider b- Limit the client´s fluids c- Prepare the client for an echocardiogram d- Document in the client´s record 170- The healthcare provider prescribes a fluid challenge of 0.9% sodium chloride 1,000 ml to be infused over 4 hours. The IV administration set delivers 10gtt/ml. How many gtt /minute should the nurse regulate the infusion? (Enter numeric value only. If rounding is required, round to the nearest whole number) 42gtt/min 171- When conducting diet teaching for a client who is on a postoperative full liquid diet, which foods should the nurse encourage the client to eat? a- Creamy peanut butter b- Vegetable juice c- Clear beef broth d- Canned fruit cocktail e- Vanilla frozen yogurt 171- A young adult woman visits the clinic and learns that she is positive for BRCA1 gene mutation and asks the nurse what to expect next. How should the nurse respond? a- Explain that counseling will be provided to give her information about her cancer risk b- Gather additional information about the client‟s family history for all types of cancer. c- Offer assurance that there are a variety of effective treatments for breast cancer. d- Provide information about survival rates for women who have this genetic mutation. 172- A client with myasthenia Gravis (MG) is receiving immunosuppressive therapy. Review recent laboratory test results show that the client‟s serum magnesium level has decreased below the normal range. In addition to contacting the healthcare provider, what nursing action is most important? a- Check the visual difficulties b- Note most recent hemoglobin level c- Assessed for he and Hand joint pain d- Observe rhythm on telemetry monitor173- Which client should the nurse assess frequently because of the risk for overflow incontinence? A client a- Who is bedfast, with increased serum BUN and creatinine levels b- With hematuria and decreasing hemoglobin and hematocrit levels c- Who has a history of frequent urinary tract infections d- Who is confused and frequently forgets to go to the bathroom 174- A male client with cirrhosis has jaundice and pruritus. He tells the nurse that he has been soaking hot baths at night no relief of his discomfort. What action should the nurse take? a- Explain that the symptoms are caused by liver damage and cannot be relieved b- Obtain a PRN prescription for an analgesic that the client can use for symptom relief c- Encourage the client to use cooler water and apply calamine lotion after soaking d- Suggest that the client take brief showers and apply oil-based lotion after showering 175- An older client comes to the clinic with a family member. When the nurse attempts to take the clients health history, the client does not respond to questions in a clear manner. What action should the nurse implement first? a- Assess the surroundings for noise and distractions b- Provide a printed health care assessment form c- Defer the health history until the client is less anxious d- Ask the family member to answer the question 176- What intervention should the nurse implement during the administration of a vesicant chemotherapeutic agent via an IV site in the client's arm? a- Assess IV site frequently for signs of extravasation. b- Monitor capillary refill distal to the infusion site c- Apply a topical anesthetic at the infusion site for burning d- Explain that temporary burning at the IV site may occur 177- A mother brings her child, who has a history of asthma, to the emergency room. The child is wheezing and speaking one world between each breath. The child is anxious, tachycardia, and has labored respirations. Which assessment is most important for the nurse to obtain? a- Type of inhaler the child typically uses on a regular basis b- Frequency that the child uses a rescue inhaler during the week c- Last dose and type rescue inhaler used by the child d- Type of allergen exposure or trigger for the current episode 178- A client takin clopidogrel reports to the onset of diarrhea. Which nursing action should the nurse implement first? a- Observe the appearance of the stool b- Assess the elasticity of the client´s skinc- Review the client´s laboratory d- Auscultate the client´s bowel sounds 179- A 12-year-old client who had an appendectomy two days ago is receiving 0.9%normal saline at 50ml/hour. The client´s urine specific gravity is 1.035. What action should the nurse implement? a- Encourage popsicles and fluids of choice b- Evaluate postural blood pressure measurements c- Obtain a specimen for urinalysis d- Assess bowel sounds in all quadrant 180- The nurse is assessing a client with pulmonary edema who is reporting two pillow orthopnea and paroxysmal nocturnal dyspnea. The nurse identifies rapid shallow respirations and use of accessory muscles. Which action should the nurse include in the client´s plan of care? a- Administer the prescribed amiodarone (Cordarone) immediately b- Institute a daily restriction while the client is in the hospital c- Perform a blood draw for serum Troponin, CK, and CK-MB levels d- Place the client in the Trendelenburg position to facilitate perfusion 181- Following and gunshot wound, an adult client a hemoglobin level of 4 grams/dl (40 mmol/L SI). The nurse prepares to administer a unit of blood for an emergency transfusion. The client has AB negative blood type and the blood bank sends a unit of type A Rh negative, reporting that there is not type AB negative blood currently available. Which intervention should the nurse implement? a- Transfuse Type A negative blood until type AB negative is available. b- Recheck the client‟s hemoglobin, blood type and Rh factor. c- Administer normal saline solution until type AB negative is available d- Obtain additional consent for administration of type A negative blood 182- The healthcare provider prescribes potassium chloride 25 mEq in 500 ml D_5W to infuse over 6 hours. The available 20 ml vial of potassium chloride is labeled, “10 mEq/5ml.” how many ml of potassium chloride should the nurse add the IV fluid? (Enter numeric value only. If is rounding is required, round to the nearest tenth.) 12.5 183- A young adult male who is being seen at the employee health care clinic for an annual assessment tell the nurse that his mother was diagnosed with schizophrenia when she was his age and that life with a schizophrenic mother was difficulty indeed. Which response is best for the nurse to provide? a- Ask the client if he is worried about becoming schizophrenic at the age his mother was diagnosed. b- Encourage the client to seek genetic counseling to determine his risk for mental illness.c- Informed the client that his mother schizophrenic has affected his psychological development. d- Tell the client that mental illness has a familial predisposition so he should see a psychiatrist. 184- After 2 days treatment for dehydration, a child continues to vomit and have diarrhea. Normal saline is infusing, and the child´s urine output is 50ml/hour. During morning assessment, the nurse determines that the child is lethargic and difficult to arouse. Which action should the nurse implement? a- Perform s finger stick glucose b- Increase the IV fluid flow rate c- Review 24-hour intake and output d- Obtain arterial blood gases 185- The healthcare provider prescribed furosemide for a 4-year-old child who has a ventricular septal defect. Which outcome indicates to the nurse that this pharmacological intervention was effective? a- Urine specific gravity change from 1.021 to 1.031 b- Daily weight decrease of 2 pounds (0.9Kg) c- Blood urea nitrogen (BUN) increase from 8 to 12 mg/dl (2.9 to 4.3) d- Urinary output decrease of 5 ml/hour 186- A client with a history of dementia has become increasingly confused at night and is picking at an abdominal surgical dressing and the tape securing the intravenous (IV) line. The abdominal dressing is no longer occlusive, and the IV insertion site is pink. What intervention should the nurse implement? a- Replace the IV site with a smaller gauge. b- Redress the abdominal incision c- Leave the lights on in the room at night. d- Apply soft bilateral wrist restraints. 187- A young adult client was admitted 36 hours ago for a head injury that occurred as the result of a motorcycle accident. In the las 4 hours, the client´s urine output has increased to over 200 ml-hour. Before reporting the finding to the healthcare provider, which intervention should the nurse implement? a- Obtain capillary blood samples for glucose every 2 hours b- Measure oral secretions suctioned during las 4 hours c- Evaluate the urine osmolality and the serum osmolality values d- Obtain blood pressure and assess for dependent edema 188- A clinical trial is recommended for a client with metastatic breast cancer, but she refuses to participate and tells her family that she does not wish to have further treatments. The client‟s son and daughter ask the nurse to try and convince their mother to reconsider this decision. How should the nurse respond? a- Ask the client with her children present if she fully understands the decision she has made. b- Discuss success of clinical trials and ask the client to consider participating for one month. c- Explain to the family that they must accept their mother‟s decision. d- Explore the client‟s decision to refuse treatment and offer support 189- The nurse is preparing a client who had a below-the-knee (BKA) amputation for discharge to home. Which recommendations should the nurse provide this client? (Select all that apply) a- Inspect skin for redness b- Use a residual limb shrinker c- Apply alcohol to the stump after bathing d- Wash the stump with soap and water e- Avoid range of motion exercises 190- When obtaining subjective data from a client, what intervention should the nurse implement first? a- listen attentively b- establish rapport c- list problems d- clarify inferences 191- A 46-year-old male client who had myocardial infarction 24-hours ago comes to the nurse‟s station fully dressed and wanting to go home. He tells the nurse that he is feeling much better at this time. Based on this behavior, which nursing problem should the nurse formulate? a- Anxiety related to treatment plan b- Deficient knowledge of lifestyle changes c- Ineffective coping related to denial d- Decisional conflict due to stress 192- A nurse working on an endocrine unit should see which client first? a- An adolescent male with diabetes who is arguing about his insulin dose. b- An older client with Addison‟s disease whose current blood sugar level is 62mg/dl (3.44 mmol/l). c- An adult with a blood sugar of 384mg/dl (21.31mmol/l) and urine output of 350 ml in the last hour. d- A client taking corticosteroids who has become disoriented in the last two hours.193- The nurse implements a tertiary prevention program for type 2 diabetes in a rural health clinic. Which outcome indicates that the program was effective? a- Clients who developed disease complications promptly received rehabilitation b- More than 50% of at-risk clients were diagnosed early their disease process c- Only 30%of clients did not attend self-management education sessions d- Average client scores improved on specific risk factor knowledge test 194- An older adult resident of a long-term care facility has a 5-year history of hypertension. The client has a headache and rate the pain 5 on a pain scale 0 to 10. The client‟s blood pressure is currently 142/89. Which interventions should the nurse implement? (Select all that apply) a- Administer a daily dose of lisinopril as scheduled. b- Assess the client for postural hypotension. c- Notify the healthcare provider immediately d- Provide a PRN dose of acetaminophen for headache e- Withhold the next scheduled daily dose of warfarin. 195- The healthcare provider changes a client‟s medication prescription from IV to PO administration and double the dose. The nurse notes in the drug guide that the prescribed medication, when given orally, has a high first-pass effect and reduces bioavailability. What action should the nurse implement a- Continue to administer the medication via the IV route b- Give half the prescribed oral dose until the provider is consulted. c- Administer the medication via the oral route as prescribed. d- Consult with the pharmacist regarding the error in prescription. 196- The charge nurse of a cardiac telemetry unit is assigning client care to a registered nurse (RN) and a practical nurse (PN). Which client should be assigned to the RN? a- One day after a permanent pacemaker insertion, a client´s telemetry monitor shows a pacer rhythm b- Two hours after undergoing a cardioversion, a client´s telemetry monitor shows a normal sinus rhythm c- A client started on carvedilol the previous day for heart failure has controlled atrial fibrillation d- Four hours after admission, a client with syncope shows complete heart block on the telemetry monitor. 197- While caring for a client with bilateral chest tubes, the bubbling in the water-seal chamber of the right chest tube stops. What action is most important for the nurse to take? a- Check the chest tube connections to the water-seal container b- Replace the water-seal collection containerc- Increase the amount of wall suction connected to right chest tube d- Milk the tubing connected to the right chest tube 198- An adult client presents to the clinic with a large draining ulcers on both lower legs that are characteristic of Kaposi´s Sarcoma lesions. The client is accompanied by two family members. What action should the nurse take? a- Obtain a blood sample to determine if the client is HIV positive b- Send family to the waiting area while the clients history is taken c- Complete a head to toe assessment to identify other signs of HIV d- Ask the family members to wear gloves when touching the client 199- The nurse is managing the care of a client with Cushing‟s syndrome. Which interventions should the nurse delegate to the unlicensed assistive personnel (UAP)? (Select all that apply) a- Evaluate the client for sleep disturbances b- Weigh the client and report any weight gain. c- Report any client complaint of pain or discomfort. d- Note and report the client‟s food and liquid intake during meals and snacks. e- Assess the client for weakness and fatigue 200- The nurse is caring for a group of clients with the help of a practical nurse (PN). Which nursing actions should the nurse assign to the PN? (Select all that apply.) A. Administer a dose of insulin per sliding scale for a client with type 2 diabetes mellitus (DM). B. Obtain postoperative vital signs for a client one day following unilateral knee arthroplasty C. Perform daily surgical dressing change for a client who had an abdominal hysterectomy D. Initiate patient controlled analgesia (PCA) pumps for two clients immediately postoperative E. Start the second blood transfusion for a client twelve hours following a below knee amputation 201-A client is admitted to a medical unit with the diagnosis of gastritis and chronic heavy alcohol abuse. What should the nurse administered to prevent the development of Wernicke's syndrome? a- Lorazepam (Ativan) b- Famotidine (Pepcid) c- Thiamine (Vitamin B1) d- Atenolol (Tenormin) 202- What information should the nurse include in the discharge teaching plan of a client with low back pain who is taking cyclobenzaprine to control muscle spasms? a- Take this medications on an empty stomach b- Avoid using heat or ice to injured muscles while taking this medication c- Use cold and allergy medications only as directed by a healthcare d- Discontinue all nonsteroidal anti-inflammatory medications203- A client with chronic kidney disease has an arteriovenous (AV) fistula in the left forearm. Which observation by the nurse indicates that the fistula is patent? a- Distended, tortuous veins in the left hand b- The left radial pulse is 2+ bounding c- Auscultation of a thrill on the left forearm d- Assessment of a bruit on the left forearm 204- When implementing a disaster intervention plan, which intervention should the nurse implement first? a- Initiate the discharge of stable clients from hospital units b- Identify a command center where activities are coordinated c- Assess community safety needs impacted by the disaster d- Instruct all essential off-duty personnel to report to the facility. 205- A client with eczema is experiencing severe pruritus. Which PRN prescriptions should the nurse administer? (Select all that apply) a- Topical corticosteroid. b- Topical scabicide. c- Topical alcohol rub. d- Transdermal analgesic. e- Oral antihistamine 206- A lethargic one-hour-old infant is brought to the nursery for further assessment. In what order should the nurse assess this infant? ( Arrange the first on the top, and the last item on the bottom) Heel stick Respirations Heart rate Temperature207- A male client approaches the nurse with an angry expression on his face and raises his voice, saying, ´´ My roommate is the most selfish, self –centered, angry person I have ever met. If the loses his temper one more time with me, I am going to punch him out‟‟ The nurse recognizes that the client is using which defense mechanism? a- Denial b- Splitting c- Projection d- Rationalization 208- The nurse is preparing to administer an oral antibiotic to a client with unilateral weakness, ptosis, mouth drooping and, aspiration pneumonia. What is the priority nursing assessment that should be done before administering this medication? a- Ask the client about soft foods preferences b- Auscultate the client‟s breath sounds c- Obtain and record the client‟s vital signs d- Determine which side of the body is weak. 209- The nurse is admitting a client from the post anesthesia unit to the postoperative surgical care unit. Which prescription should the nurse implement first? a- Advance from clear liquids as tolerated b- Straight catheterization if unable to void c- Cefazolin 1 gram IVPQ q6 hours d- Complete blood cell count (CBC) in AM 210- On admission to the Emergency Department, a female client who was diagnosed with bipolar disorder 3 years ago reports that this morning she took a handful of medications and left a suicide note for her family. Which information is most important for the nurse to obtain? a- Which family member has the clients suicide note b- When the client las took drugs for bipolar disorder c- What drugs the client used for the suicide attempt d- Whether the client ever attempted suicide in the past 211- A client present s to the healthcare provider with fatigue, poor appetite, general malaise, and vague joint pain that improves mid- morning. The client has been over the counter ibuprofen for several months. The healthcare provide makes an initial diagnosis of rheumatoid arthritis (RA). Which laboratory test should the nurse report to the healthcare provider ? (Select all apply) a- Sedimentation rate b- White blood cell count c- Anti – CCP antibodies d- Activated Clotting Timee- C- reactive protein 212- A female client is admitted for diabetic crisis resulting from inadequate dietary practices. After stabilization, the nurse talks to the client about her prescribed diet. What client characteristic is most import for successful adherence to the diabetic diet? a- Knows that insulin must be given 30 min before eating b- Frequently eats fruits and vegetables at meals and between meals/ c- Has someone available who can prepare and oversee the diet d-Demonstrates willingness to adhere to the diet consistently 213- An infant is receiving penicillin G procaine 220,000 units IM. The drug is supplied as 600,000 units/ml. How many ml should the nurse administer? (Enter numeric value only. If rounding is required, round to the nearest tenth) 0.4 Rationale: Calsulate using the formula, desired dose (220,000 units) over dose on hand (600,000 units) x the volume of the available dose (1 ml). 220,000 / 600,000 x 1 ml = 0.36 = 0.4 ml 214- A client with chronic alcoholism is admitted with a decreased serum magnesium level. Which snack option should the nurse recommend to this client? a- Cheddar cheese and crackers. b- Carrot and celery sticks. c- Beef bologna sausage slices. d- Dry roasted almonds. 215-A male client is returned to the surgical unit following a left nephrectomy and is medicated with morphine. His dressing has a small amount of bloody drainage, and a Jackson-Pratt bulb surgical drainage device is in place. Which interventions is most important for the nurse to include in this clients plan of care? a- Monitor urine output hourly. b- Assess for back muscle aches c- Record drainage from drain d- Obtain body weight daily 216- The nurse discontinues a continuous IV heparin infusion for a male client on strict bedrest, and is now preparing to administer the clients first dose of enoxaparin. Prior to giving this subcutaneous injection, which assessment finding requires additional intervention by the nurse? a- Current lab report indicates an Aptt at 1.5 times the client control b- The client states that his right calf is aching, and wants pain medicationc- The spouse is assisting the client who is shaving with an electric razor d- Several bruised areas are noted on the clients upper extremities bilaterally 217- The nurse is caring for a client who is receiving continuous ambulatory peritoneal dialysis (CAPD) and notes that the output flow is 100ml less than the input flow. Which actions should the nurse implement first? a- Continue to monitor intake and output with next exchange b- Check the clients blood pressure and serum bicarbonate c- Irrigate the dialysis catheter d- Change the clients position 218- When administer ceftriaxone sodium intravenously to a client before surgery, which assessment finding requires the most immediate intervention by the nurse? a- Headache b- Pruritis c- Nausea d- Stridor 219- After assessing an older with a suspected cerebrovascular accident (CVA), the nurse documents the client´s right upper arm weakness and slurred speech. When the client complains of a severe headache and nausea, the clients neurological assessment remains unchanged. Which action should the nurse implement first? a- Administer an oral analgesic with antiemetic b- Collect blood for coagulation times c- Send the client for a computed tomography scan the brain d- Obtain a history of medication use, recent surgery, or injury 220- The nurse observes an unlicensed assistive personnel (UAP) who is preparing to provide personal care for a client who requires contact precautions. The UAP has applied a gown and gloves and secured the tops of the gloves over the gown sleeves. What action should the nurse take? a- Confirm that the gown is tied securely at the neck and waist b- Remind the UAP to wash hands frequently while in the room c- Assist the UAP with application of the face mask or face shield d- Help the UAP reposition the gown sleeve over the gloves edges 221- An adult client with severe depression was admitted to the psychiatric unit yesterday evening. Although the client ran one year ago, his spouse states that the client no longer runs, bur sits and watches television most of the day. Which is most important for the nurse to include in this client‟s plan of care for today?a- Assist client in identifying goals for the day. b- Encourage client to participate for one hour in a team sport. c- Schedule client for a group that focuses on self-esteem. d- Help client to develop a list of daily affirmations. 222- An older adult male who had an abdominal cholecystectomy has become increasingly confused and disoriented over the past 24 hours. He is found wandering into another client‟s room and is return to his room by the unlicensed assistive personnel (UAP). What actions should the nurse take? (Select all that apply). a- Apply soft upper limb restrains and raise all four bed rails b- Report mental status change to the healthcare provider c- Assess the client‟s breath sounds and oxygen saturation d- Assign the UAP to re-assess the client‟s risk for falls e- Review the client‟s most recent serum electrolyte values 223- A client with bleeding esophageal varices receives vasopressin (Pitressin) IV. What should the nurse monitor for during the IV infusion of this medication? a- Vasodilatation of the extremities b- Chest pain and dysrhythmia c- Hypotension and tachycardia d- Decreasing GI cramping and nausea 224- A client with a large pleural effusion undergoes a thoracentesis. Following the procedure, which assessment finding warrants immediate intervention by the nurse? a- The client has asymmetrical chest wall expansion b- The clients complain of pain at the insertion site c- The client chest‟s x-ray indicates decreased pleural effusion d- The client‟s arterial blood gases are pH 7.35, PaO2 85, Pa CO2 35, HCO3 26 225- A client with multiple sclerosis is receiving baclofen 15 mg PO three times daily. The drug is available in 10 mg tablets. How many tablets should the nurse administer in a 24 hour period? (Enter numeric Value only. If rounding is required, round to the nearest tenth) 4.5 226- The nurse is completing a neurological assessment on a client with a closed head injury. The Glasgow Coma Scale (GCS) score was 13 on admission. It is now assessed at 6. What is the priority nursing intervention based on the client´s current GCS? a- Notify the healthcare provider of the GCS score b- Prepare the family for the client‟s imminent death c- Monitor the client q1 hour for changes in the GCS score d- Begin cardiopulmonary resuscitation (CPR)227- Two days after an abdominal hysterectomy, an elderly female with diabetes has a syncopal episode. The nurse determines that her vital signs within normal limits, but her blood sugar is 325mg/dL or 18.04 mmol/L (SI). What intervention should the nurse implement first? a- Administer regular insulin per sliding scale b- Cancel the client‟s dinner tray c- Give the client 4 ounces (120mL) or orange juice d- Administer next scheduled dose of metformin 228- When washing soiled hands the nurse first wets the hands and applies soap. The nurse should complete additional actions in which sequence? (Arrange from first action on top to last action on botton.) RESPONDIDA Rub hands palm to palm Interlace the fingers Dry hands with paper towel turn off water faucet 229- A male client with hypertension, who received new antihypertensive prescriptions at his last visit returns to the clinic two weeks later to evaluate his blood pressure (BP). His BP is 158/106 and he admits that he has not been taking the prescribed medication because the drugs make him “feel bad”. In explaining the need for hypertension control, the nurse should stress that an elevated BP places the client at risk for which pathophysiological condition? a- Feed the client a snack b- Empty the urinary drainage bag c- Offer the client oral fluids d- Stroke secondary to hemorrhage 230- A client who underwent an uncomplicated gastric bypass surgery is having difficult with diet management. What dietary instruction is most important for the nurse to explain to the client? a- Chew food slowly and thoroughly before attempting to swallow b- Plan volume-controlled evenly-space meal thorough the day c- Sip fluid slowly with each meal and between meals d- Eliminate or reduce intake fatty and gas forming food231- Four clients arrive on the labor and delivery unit at the same time. Which client should the nurse assess first? a- A 38-week primagravida who reports contractions occurring every 10 minutes b- A 39-week primagravida with biophysical profile score of 5 out of 8. c- A 41-week multigrada who is scheduled induction of labor today d- A 36-week multigrada with a prescription for serial blood pressure 232- The nursing staff on a medical unit includes a registered nurse (RN), practical nurse (PN), and an unlicensed assistive personnel (UAP). Which task should the charge nurse assign to the RN? a- Transport a client who is receiving IV fluid to the radiology department b- Administer PRN oral analgesics to a client with a history of chronic pain. c- Supervise a newly hired graduated nurse during an admission assessment d- Complete ongoing focused assessments of a client with wrist restraints 233- The nurse enters the room of a client with Parkinson‟s disease who is takin carbidopalevodopa. The client is arising slowly from the chair while the unlicensed assistive personnel (UAP) stands next to the chair. What action should the nurse take? a- Demostrate how to help the client move more efficiency b- Offer a PRN analgesic to reduce painful movement c- Affirm that the client should arise slowly from the chair d- Tell the UAP assist the client moving more quickly 234. A young adult female presents at the emergency center with acute lower abdominal pain. Which assessment finding is most important for the nurse to report to the healthcare provider? a- Pain scale rating at 9 on a 0-10 scale b- Last menstrual period was 7 weeks ago c- Reports white curdy vaginal discharge d-History of irritable bowel syndrome IBS 235- A client with renal disease seems axious and presents with the onset of shortness of breath, lethargy, edema, and weight gain. Which action should the nurse implement first? a- Determine serum potassium level b- Calculate the clients daily fluid intake c- Assess client for signs of vertigo d- Review the client‟s pulse oximetry reading 236- A nurse who is working in the emergency department triage area is presented with four clients at the same time. The client presented with which symptoms requires the most immediate intervention by the nurse? a- Low-grade fever, headache, and malaise for the past 72 hours b- Unable to bear weight on the left foot, with the swelling and bruising c- Chest discomfort one hour after consuming a large, spicy meald- One-inch bleeding laceration on the chain of the crying five-year-old 237- The nurse changes a Pleuravac® chest drainage system for a client whose chest collection unit is filled to capacity. After unclamping the attached chest tube, the nurse turns on the suction to 20 cm water pressure, and 125 ml of serosanguineous drainage is obtained. Which location on the Pleuravac® indicates this volume of drainage from the chest tube? (Click the chosen location. 238- A client with a pneumothorax is admitted for insertion of a chest tube that is attached to suction with a chest drainage system. The nurse verifies that the suction level is at 20 cm water pressure. Which location on the chest drainage system indicates this finding? 239- The nurse determines that 850 ml of serosanguineous drainage is in the Pleuravac chest drainage system of a client who is 12 hours postoperative for the removal of lung tumor. Which location on the chest collection unit indicates the finding? (Click the chosen location. To change, click on the new location.)240- The nurse has completed the diet teaching of a client who is being discharged following treatment of a leg wound. A high protein diet is encouraged to promote wound healing. Which lunch choice by the client indicates that the teaching was effective? a- A tuna fish sandwich with chips and ice cream. b- A peanut butter sandwich with soda and cookies. c- A salad with three kinds of lettuce and fruit. d-Vegetable soup, crackers, and milk. 241- During a postpartum assessment of a client who is 5 hours post vaginal delivery, the nurse determines the fundus is 3 finger breadths above the umbilicus and positioned to the client‟s side. Which action should the nurse implement first? a- Massage the fundus until firm. b- Encourage the client to void. c- Provide additional oral replacement fluids. d-Catheterize for residual urinary volume. 242-The nurse is inspecting the thoracic region of an adolescent client. Which finding is consistent with scoliosis. a- Asymmetry. b-Masses c-Erythema d-Barrel shape. 243- While changing the dressing of a client who is immobile, the nurse notices the boundary of the wound has increased. Before reporting this finding to the healthcare provider, the nurse should evaluate which of the client‟s laboratory values? a- Serum potassium and sodium levels. b- C-reactive protein level. c- Platelet count. d- Neutrophil count. 244- A client who is admitted to the medical unit with fluid volume overload has 2+ dependent edema in the lower extremities and a serum sodium level of 115mEq/L. Which assessment should the nurse complete next?a- Skin turgor. b- Mental status. c- Bowel sound. d-Urinary output. 245- A client who will be going to surgery states no known allergies to any medications. What is the most important nursing action for the nurse to implement next? a- Assess client‟s knowledge of an allergic response b- Record “no known drug allergies” on preoperative checklist c- Flag “no known drug allergies” on the front of the chart. d- Assess client‟s allergies to non-drug substances. 246- A nurse who works in the nursery is attending the vaginal delivery of a term infant. What action should the nurse complete prior to leaving the delivery room? a- Obtain the infant‟s vital signs. b- Observe the infant latching onto the breast c- Administer vitamin K injection d- Place the ID bands on the infant and mother. 247- A client who had a recurrent episode of urolithiasis is admitted for acute flank pain in the upper right abdominal quadrant, diaphoresis, and vomiting. After medicating the client for pain, the nurse strains the client’s urine and collects granules and stones. Which action is most important for the nurse to implement next? a- Send the calculi for laboratory analysis of stone composition. b- Evaluate client for persistent pain c- Asses clarity of urine d- Encourage high fluid intake to produce urine output of 2L/day. 248- The nurse assesses a client one hour after starting a transfusion of packed red blood cells and determines that there are no indications of a transfusion reaction. What instructions should the nurse provide the unlicensed assistive personnel (UAP) who is working with the nurse? a- Continue to measure the client‟s vital signs every thirty minutes until the transfusion is complete b- Since a reaction did not occur, the priority is to maintain client comfort during the transfusion c- Monitor the client carefully for the next three hours and report the onset of a reaction immediately. d- Notify the nurse when the transfusion has finished, so further client assessment can be done.249- To reduce the risk of symptoms exacerbation for a client with multiple sclerosis (MS), which instructions should the nurse include in the client‟s discharge plan? (Select all that apply). a- Practice relaxation exercises b- Limit fluids to avoid bladder distention c- Space activities to allow for rest periods d- Avoid persons with infections e- Take warm baths before starting exercise 250- The nurse notes that an older adult client has a moist cough that increase in severity during and after meals. Based on this finding, what action should the nurse take. a) Encourage client to deep breathing exercises daily. b) Offer the client additional clear frequently. c) Collect a sputum specimen immediately. d) Request a consultation to confirm dysphagia 260- An older client is having photocoagulation for macular degeneration. What intervention should the nurse implement during the post procedure care in the outpatient surgical unit? a- Apply bilateral eye patches while sleeping b- Use a white board to communicate ideas c- Arrange food on the plate in clockwise order. d- Verbally identify self when entering the room 261- The nurse is assessing a client with left-sided heart failure who reports nocturia and dyspnea. The nurse identifies pulsus alternans and crackles in all lung fields. Which action is best to include in the client‟s plan of care? a- Begin client education about positive inotropic medication. b- Place the client in Trendelenburg position c- Prepare the client for an emergency cardiac catherization d- Monitor serum Troponin, CK, and CK – MB levels. 262- The nurse plans to contact the healthcare provider regarding a client‟s need for a belt restraint. What information is most important to report to the healthcare provider? a- The presence and location of any pressure ulcers. b- Measures already taken to maintain client safety c- Any special mattresses on the client‟s bed. d- Current vital signs and oxygen saturation 263- The nurse is teaching a male adolescent recently diagnose with type 1 diabetes mellitus (DM) about self-injecting insulin. Which approach is best for the nurse to use to evaluate the effectiveness of the teaching? a- Observe him as he demonstrates the self-injection technique to another diabetic adolescent b- Ask the adolescent to describe his level of comfort with injecting himself with insulin c- Review his glycosylated hemoglobin level 3 months after the teaching session. d- Have the adolescent list the procedural steps for safe insulin administration.264- A client with leukemia who is receiving a myleosupressive chemotherapy has a platelet count of 25,000/mm3. Which intervention is most important for the nurse to include in this client‟s plan of care? a- Monitor for signs of activity intolerance b- Require visitors to wear respiratory masks c- Asses urine and stool for occult blood d-Obtain client‟s temperature q4 hours. 265- Three hours following a right carotid endarterectomy, the nurse notes a moderate amount of bloody drainage on the client‟s dressing. Which additional assessment finding warrants immediate intervention by the nurse. a- Sore throat when swallowing b- Tongue deviation to the left c- Palpable temporal pulses d- Temperature of 99.2F (37.3 C) 266- A client is admitted with pyelonephritis and cultures reveal Escherichia coli infection. The client is allergic to penicillins, and the healthcare provider prescribed vancomycin IV. The nurse should plan to carefully monitor the client for which finding during IV administration? a- Tissue sloughing on extravasation b- Elevated blood pressure and heart rate c- Tinnitus and vertigo d- Erythema of facial neck and chest. 267- A young female adult wander into the Emergency Department. She is disheveled and confused and states, "My date must have put something in my drink. He took my car, and I think he raped me. I don't exactly remember, but I know he hurt me." How should the nurse respond? a- Did you try to resist or fight back when you were attacked? b- He hurt you? What makes you think you were raped? c- It is ok to cry, but the first we need to take care of your injuries d- Yes, I can see. Tell me more about what you remember 268- While assessing an older client‟s fall risk, the client tell the nurse that they live at home alone and has never fallen. What action should the nurse take? a- Place the client on a high fall risk protocol because of his age b- Continue to obtain the client data needed to complete the fall risk survey c- Inform the client that falls occur more than often in the hospital than at home d- Record a minimal risk for falls, documenting the client statement. 269- A female client with fibromyalgia asks the nurse to arrange for hospice care to help her manage the severe, chronic pain. Which interdisciplinary team member should the nurse consult to assist the client? a- Contact a hospice nurse for an evaluation b- Arrange an appointment with a pain specialist c- Ask for a consultation with a psychologist d- Form an interdisciplinary team for evaluation 270- A client with pneumonia has arterial blood gases levels at: pH 7.33; PaC02 49 mm/ Hg; HC03 25mEq/L; Pao., 95. What intervention should the nurse implement based on these results? a- Instruct the client to breathe into a paper bag b- Initiate oxygen administration at 2 to 3/L per nasal cannula c- Institute coughing and deep breathing protocols d- Prepare to administer sodium chloride fluids 271- Which nursing responsibility is related to health promotion and teaching for the client with rheumatoid arthritis? a- Application of heat and cold therapy b- Immobilization of affected joints c- Avoidance of foods containing purine d- Prevention through nutrition and exercises 272- Which class of drugs is the only source of a cure for septic shock? a- Antihistamine b- Antiinfectives c- Anticholesteremics d- Antihypertensives 273- Prior to insertion of indwelling urinary catheter, what client information is most important for the nurse to obtain. a- Client allergies to antiseptic solutions b- Color, clarity and odor of urine c- Client‟s ability to increase fluid intake d- Previous history of urinary tract infections 274- The charge nurse observes a new nurse preparing to administer an intramuscular injection. The new nurse is attaching a 24-gauge needle. What action should the charge nurse take? a- No corrective action is needed by the nurse b- Direct the nurse to substitute a larger bore needle c- Tape 4x4 gauze dressing to the site d- Instruct the nurse to use water with a 5% dextrose (D5 W) 275- An older client with a history of pernicious anemia has developed ataxia and paresthesia. In planning care, which nursing intervention has the highest priority?a- Instruct about healthy diet choices b- Provide assistance with ambulation c- Keep the head of the bed elevated d- Offer a PRN sleep aid at night 276- An adult female client tells the nurse that though she is afraid her abusive boyfriend might one-day kill her, she keeps hoping that he will change. What action should the nurse take first? a- Report the finding to the police department b- Discuss treatment options for abusive partners c- Determine the frequency and type of client‟s abuse d- Explore client‟s readiness to discuss the situation. 278- A young adult female college student visits the health clinic in early winter to obtain birth control pills. The clinic nurse asks if the student has received an influenza vaccination. The student stated she did not receive vaccination because she has asthma. How should the nurse respond? a- Offer to provide the influenza vaccination to the student while she is at the clinic b- Encourage the student to obtain a vaccination prior to the next influenza season. c- Confirm that a history of asthma can increase risks associated with the vaccine. d- Advise the student that the nasal spray vaccine reduces side effects for people with asthma. 279- Dopamine 5mcg/kg/ minute IV is prescribed for a client who weighs 132 pounds. The pharmacy dispenses a 500 ml IV solution of 0.9 % normal saline with dopamine 1600 mg. The nurse should program the infusion pump to deliver how many ml/hour? ( Enter numeric value only. If rounding is required, round to the nearest tenth.) 5.6 280- The nurse is explaining the need to reduce salt intake to a client with primary hypertension. What explanation should the nurse provide? a- High salt can damage the lining of the blood vessels b- Too much salt can cause the kidneys to retain fluid c- Excessive salt can cause blood vessels to constrict d- Salt can cause information inside the blood vessels 281- A client with a newly diagnosed type 2 diabetes mellitus, awakens in the morning with a temperature of 101.6 F (38.7C). The client calls the clinic nurse and reports taking the morning dose of metformin after obtaining a morning fasting blood glucose of 325mg/dl (18mmol/L). Which instruction should the nurse provide to the client? a- Monitor blood glucose levels more frequently b- Watch closely for clinical manifestations of hypoglycemia c- Begin a routine sliding scale for lispro insulind- Increase the daily dose of metformin. 282- When preparing for a bone marrow aspiration, the nurse should place the client in which position to ensure access of the aspiration site? a- Prone with the posterior iliac crest draped b- Side-lying with the posterior chest exposed c- Sitting up and leaning across the over-bed table d- Supine with a pillow under the lumbar area. 283- When admitting a client with a diagnosis of transient ischemic attack (TIA), which intervention is most important for the nurse to include in this client‟s plan of care? a- Assess bilateral breath sounds b- Initiate neurological monitoring every 2 hours c- Review client‟s daily medications d- Palpate suprapubic region for urinary retention 284- A low risk primigravida at 28-weeks gestation arrives for her regular antepartal clinic visit. Which assessment finding should the nurse consider within normal limits for this client? a- Glucosuria b- Proteinuria c- Fundal height of 22 centimeters d- Pulse increase of 10 beats/minute 285- The nurse should withhold medication if the client‟s serum potassium level is 6.2 mEq/L or mmol/L (SI)? a- Spironolactone b- Metolazone c- Furosemide d- Hydrochlorothiazide 286- A client is admitted with heart failure and left ventricular hypertrophy. Which intervention is most likely to avert development of cardiomegaly and improve myocardial contractility? a- Administer nitroglycerin (Nitro-Bid) daily. b- Provide rest periods and portable oxygen c- Maintain continuous monitoring of heart rate and rhythm d- Teach about compliance to reduce blood pressure 287- An adolescent girl with anorexia nervosa is within 10 pounds of her normal weight and is being discharge from the mental health unit with a prescription for fluoxetine (prozac). Which instruction is most important for the nurse to provide the parent listen for the daughter expressions of wanting to harm herself a- Listen for the adolescent‟s expressions of wanting to harm themselvesb- Check the adolescent‟s mouth to ensure that they had swallowed fluoxetine c- Observe their adolescent weight themselves at the same time every day d- Encourage activities that allow their adolescent to exhibit control 288- A client who was admitted yesterday with severe dehydration is complaining of pain a 24- gauge IV with normal saline is infusing at a rate of 150 ml/hour. Which intervention should the nurse implement first? a- Establish the second IV site b- Asses the IV for blood return c- Stop the normal saline infusion. d- Discontinue the 24-gauge IV 289- A client at 12- weeks gestation is admitted to the antepartum unit with a diagnosis of hyperemesis gravidarum. Which action is most important for the nurse to implement? a- Schedule a consult with a nutritionist b- Obtain the client‟s 24- hour diet recall c- Document mucosal membrane status d- Initiate prescribed intravenous fluids 290- A male client with a history of mitral valve prolapse is admitted because of fever and dyspnea on exertion, and is diagnosed with acute infective endocarditis. During the admission assessment, the nurse observes multiple areas of petechiae on the client‟s skin. Which interventions should the nurse include in the client‟s plan of care? (Select all that apply) a- Maintain record of fluid intake and output b- Initiate contact transmission precautions c- Report changes in pre – existing murmurs d- Monitor cardiac rhythm via telemetry e- Schedule rest periods between activities 291- The nurse is teaching a group of women about osteoporosis and exercises. The nurse should emphasize the need for which type of regular activity. a- Core strengthening b- Muscle stretching and toning c- Weight – bearing exercise d- Aerobic exercise 292- A client is admitted with complains of a racing heart, light headedness, and shortness of the breath. The cardiac monitor displays supraventricular tachycardia (SVT). Which intervention should the nurse implement first? a- Obtain a 12 electrocardiogram b- Explain importance of reducing the heart ratec- Help with synchronized cardioversion d- Administer an intravenous anxiolytic 293- The nurse is caring for an adult male client who was admitted to intensive care unit (ICU) following a motor vehicle collision in which he sustained a crushing injury to the chest. Which statement regarding a pulmonary contusion is accurate? a- Minimal observation and monitoring are required. b- This injury responds well to high Fi02 concentration c- It is associated with potential lethal injury to the client d- Occurs most often as a result of penetrating trauma 294- Several days after having a computed tomography scan (CT scan) with contrast for lower abdominal pain, an older male adult is admitted with acute kidney injury (AKI). The client‟s urinary output is 10ml/hour, his heart rate is 125 beats/minute , and basilar crackles are audible. Which laboratory value in the client‟s electronic record requires intervention by the nurse? a- Spot collection for Urine sodium 14 mEq/L (14mmol/L) b- Serum creatinine 2.1 mg/dl (186 mcmol/L) c- Blood Urea Nitrogen (BUN) 19 mg/dl (6.8 mmol/L) d- Potassium 6.8 Meq/L (6.8 mmol/L) 295- Handwashing is the most important intervention aimed at reducing the spread infection in the hospitalized population. What mechanical describes the effect that handwashing has upon the chain of infection? a- Destroys non- human reservoir b- Reduces victim‟s susceptibility c- Blocks a portal of entry d- Blocks pathogen transmission 296- What instruction should the nurse provide the parents of a 3-year-old boy with a BMI-forage at the 97th percentile? a- Your child‟s weight is in the high range, but is probably normal for his body build b- Your child is tall for his age, so be sure he gets plenty to eat to stay healthy c- Your child has very strong bones, so continue to maintain the same diet d- Your child is overweight for his age and size, so help him select more healthy foods 297- An older male client is admitted with the medical diagnosis of possible cerebral vascular accident (CVA). He has facial paralysis and cannot move his left side. When entering the room, the nurse finds the client‟s wife tearful and trying unsuccessfully to give him a drink of water. What action should the nurse take? a- Give the wife a straw to help facilitate the client‟s drinking. b- Assist the wife and carefully give the client small sips of waterc- Obtain a thickening powder before providing any more fluids. d- Ask the wife to stop and assess the client‟s swallowing reflex. 298- A client is receiving intravenous (IV) fluids by gravity infusion and exhibits signs of fluid volume overload. When assessing the client‟s IV delivery system, where should the nurse assess first? B 299- A young adult male who was in a motorcycle collision experienced a laceration of the gastrocnemius muscle. Which instruction should the nurse to provide to the practical nurse (PN) who is caring for this client? a- Perform range of motion on the affected limb b- Avoid planter flexion of the affected limb c- Elevate limb above the heart when lying in bed d- Avoid washing the limb when assisting with bathing 300- The charge nurse is planning for the shift and has a registered nurse (RN) and a practical nurse (PN) on the team. Which client should the charge nurse assign to the RN? a- An adolescent with multiple contusions due to a fall that occurred 2 days ago b- A 75-year-old client with renal calculi who requires urine straining c- A 64-year-old client who had a total hip replacement the previous day d- A 30-year-old depressed client who admits to suicide ideation 301-(VIDEO) The nurse observes an unlicensed assistive personnel (UAP) perform catheter care as seen in the video. What action should the nurse take? (Please view the video before selecting the option that applies. To repeat the video, click the play button again) a- Explain how to clean the indwelling catheter b- Affirm that the skill was completed correctly c- Review how to clean the perinea area d- Obtain urine for analysis of bacterial count302- The charge nurse is making assignments on a psychiatric unit for a practical nurse (PN) and a newly licensed registered nurse (RN). Which client should be assigned to the RN. a- An older male who tells the staff and other clients that he is Superman and can fly b- A young male with schizophrenia who says voices are telling him to kill his psychiatrist c- A middle-age client who is in the depressive phase of bipolar disease and is receiving lithium d- An adult client who has been depressed for the past several months and denies suicidal ideation 303- A client is receiving a secondary infusion of erythromycin 1 grams in 100 ml dextrose 5%in water (D5W) to be infused in 30 minutes. How many ml/hour should infusion pump? (Enter numerical value only) 200 304- The nurse is caring for a 3-month -old infant who is one day after a pylorotomy notices that the infant is restless, is exhibiting facial grimaces, and is drawing both knees to the chest. What action should the nurse take? a- Provide additional blankets to increase body temperature b- Increase IV infusion rate for rehydration c- Feed one ounce of formula to correct hypoglycemia d- Administer a prescribed analgesic for pain 304- A male client reports to the clinic nurse that he has recently experienced symptoms of polyuria, polyphagia, and polydipsia. Which laboratory findings are significant for the nurse to report to the healthcare provider? (Select all that apply) a- Hematocrit 59% (0.59SI) b- Total cholesterol 180mg/dl (4.66 mmol/L SI) c- Serum Potassium of 4.2 mEq/L (4.2 mmol/L SI) d- Fasting blood glucose of 175 mg/dl (9.7 mmol/L SI) e- White blood cell count of 18,000/ mm3 (18x10/L SI) 305- Four hours after the nurse administers interferon alfa subcutaneously to a client, the client develops a headache, muscle aches and a fever of 101.8 F (38.8 C). What action should the nurse implement? a- Document these findings as an idiosyncratic response to this medication b- Administer a prescribed PRN dose of acetaminophen for these side effects c- Observe the site where the medication was injected for signs of a local reaction d- Explain that an antihistamine may be needed in response to this allergic reaction 306- A client received a thrombolytic medication following a myocardial infarction. When the client has a bowel movement, what action should the nurse implement?a- Obtain a specimen for culture and sensitivity analysis b- Assess for fatty yellow streaks in the client‟s stool c- Send a stool sample to the lab for a guaiac test d- Observe the stool for a clay-colored appearance 307- An older adult woman is brought to the emergency department by her daughter guaiac test who reports that her mother has recently becomes confused. Further assessment indicates that the client has had the flu and has been vomiting for the past two days and she currently takes medications for diabetes, hypertension, and heart failure. Which intervention should the nurse implement first? a- Obtain a capillary blood glucose level b- List home medication names and dosage c- Insert an indwelling urinary catheter d- Establish client‟s mental status baseline 308- A mother runs into the emergency department with s toddler in her arms and tells the nurse that her child got into some cleaning products. The child smells of chemicals on hands, face, and on the front of the child's clothes. After ensuring the airway is patent, what action should the nurse implement first? a- Call poison control emergency number. b- Determine type of chemical exposure. c- Obtain equipment for gastric lavage. d- Assess child for altered sensorium. 309- In assessing an adult client with a partial rebreather mask, the nurse notes that the oxygen reservoir bag does not deflate completely during inspiration and the client‟s respiratory rate is 14 breaths / minute. What action should the nurse implement? a- Encourage the client to take deep breaths b- Remove the mask to deflate the bag c- Increase the liter flow of oxygen d-Document the assessment data 310- During a home visit, the nurse observed an elderly client with diabetes slip and fall. What action should the nurse take first? a- Give the client 4 ounces of orange juice b- Call 911 to summon emergency assistance c- Check the client for lacerations or fractures d-Asses clients blood sugar level 311- At 0600 while admitting a woman for a schedule repeat cesarean section (C-Section), the client tells the nurse that she drank a cup a coffee at 0400 because she wanted to avoid getting a headache. Which action should the nurse take first? a. Ensure preoperative lab results are available b. Start prescribed IV with lactated Ringer‟sc. Inform the anesthesia care provider d. Contact the client‟s obstetrician. 312- After placing a stethoscope as seen in the picture, the nurse auscultates S1 and S2 heart sounds. To determine if an S3 heart sound is present, what action should the nurse take first? a. Side the stethoscope across the sternum. b. Move the stethoscope to the mitral site c. Listen with the bell at the same location d. Observe the cardiac telemetry monitor 313- A 66-year-old woman is retiring and will no longer have a health insurance through her place of employment. Which agency should the client be referred to by the employee health nurse for health insurance needs? a. Woman, Infant, and Children program b. Medicaid c. Medicare d. Consolidated Omnibus Budget Reconciliation Act provision. 314- A client who is taking an oral dose of a tetracycline complains of gastrointestinal upset. What snack should the nurse instruct the client to take with the tetracycline? a. Fruit-flavored yogurt. b. Cheese and crackers. c. Cold cereal with skim milk. d. Toasted wheat bread and jelly 315- Following a lumbar puncture, a client voices several complaints. What complaint indicated to the nurse that the client is experiencing a complication? a. “I am having pain in my lower back when I move my legs” b. “My throat hurts when I swallow” c. “I feel sick to my stomach and am going to throw up” d. I have a headache that gets worse when I sit up” 316- An elderly client seems confused and reports the onset of nausea, dysuria, and urgency with incontinence. Which action should the nurse implement? a. Auscultate for renal bruits b. Obtain a clean catch mid-stream specimen c. Use a dipstick to measure for urinary ketone d. Begin to strain the client‟s urine. 317- The nurse is assisting the mother of a child with phenylketonuria (PKU) to select foods that are in keeping with the child‟s dietary restrictions. Which foods are contraindicated for this child? a. Wheat products b. Foods sweetened with aspartame. c. High fat foodsd. High calories foods. 318- Before preparing a client for the first surgical case of the day, a part-time scrub nurse asks the circulating nurse if a 3-minute surgical hand scrub is adequate preparation for this client. Which response should the circulating nurse provide? a. Ask a more experience nurse to perform that scrub since it is the first time of the day b. Validate the nurse is implementing the OR policy for surgical hand scrub c. Inform the nurse that hand scrubs should be 3 minutes between cases. d. Direct the nurse to continue the surgical hand scrub for a 5-minute duration. 319- Which breakfast selection indicates that the client understands the nurse‟s instructions about the dietary management of osteoporosis? a. Egg whites, toast and coffee. b. Bran muffin, mixed fruits, and orange juice. c. Granola and grapefruit juice d. Bagel with jelly and skim milk. 320- The charge nurse of a critical care unit is informed at the beginning of the shift that less than the optimal number of registered nurses will be working that shift. In planning assignments, which client should receive the most care hours by a registered nurse (RN)? a- A 34-year -old admitted today after an emergency appendendectomy who has a peripheral intravenous catheter and a Foley catheter. b- A 48-year-old marathon runner with a central venous catheter who is experiencing nausea and vomiting due to electrolyte disturbance following a race. c. A 63-year-old chain smoker admitted with chronic bronchitis who is receiving oxygen via nasal cannula and has a saline-locked peripheral intravenous catheter. d. An 82-year-old client with Alzheimer‟s disease newly-fractures femur who has a Foley catheter and soft wrist restrains applied 321- A mother brings her 6-year-old child, who has just stepped on a rusty nail, to the pediatrician‟s office. Upon inspection, the nurse notes that the nail went through the shoe and pierced the bottom of the child‟s foot. Which action should the nurse implement first? a. Cleanse the foot with soap and water and apply an antibiotic ointment b. Provide teaching about the need for a tetanus booster within the next 72 hours. c. have the mother check the child's temperature q4h for the next 24 hours d. transfer the child to the emergency department to receive a gamma globulin injection 322- The mother of an adolescent tells the clinic nurse, “My son has athlete‟s foot, I have been applying triple antibiotic ointment for two days, but there has been no improvement.” What instruction should the nurse provide? a. Antibiotics take two weeks to become effective against infections such as athlete‟s foot. b. Continue using the ointment for a full week, even after the symptoms disappear. c. Applying too much ointment can deter its effectiveness. Apply a thin layer to prevent maceration. d. Stop using the ointment and encourage complete drying of the feet and wearing clean socks.323- A 26-year-old female client is admitted to the hospital for treatment of a simple goiter, and levothyroxine sodium (Synthroid) is prescribed. Which symptoms indicate to the nurse that the prescribed dosage is too high for this client? The client experiences a. Palpitations and shortness of breath b. Bradycardia and constipation c. Lethargy and lack of appetite d. Muscle cramping and dry, flushed skin 324- A client with a history of heart failure presents to the clinic with a nausea, vomiting, yellow vision and palpitations. Which finding is most important for the nurse to assess to the client? a. Determine the client‟s level of orientation and cognition b. Assess distal pulses and signs of peripheral edema c. Obtain a list of medications taken for cardiac history. d. Ask the client about exposure to environmental heat. 325- The healthcare provider prescribes an IV solution of isoproterenol (Isuprel) 1 mg in 250 ml of D5W at 300 mcg/hour. The nurse should program the infusion pump to deliver how many ml/hour? (Enter numeric value only.) 75 Rationale: Convert mg to mcg and use the formula D/H x Q. 300 mcg/hour / 1,000 mcg x 250 ml = 3/1 x 25 = 75 ml/hour 326- The pathophysiological mechanisms are responsible for ascites related to liver failure? (Select all that apply) a. Bleeding that results from a decreased production of the body‟s clotting factors b. Fluid shifts from intravascular to interstitial area due to decreased serum protein c. Increased hydrostatic pressure in portal circulation increases fluid shifts into abdomen d. Increased circulating aldosterone levels that increase sodium and water retention e. Decreased absorption of fatty acids in the duodenum leading to abdominal distention. 327- The nurse is auscultating a client‟s heart sounds. Which description should the nurse use to document this sound? (Please listen to the audio first to select the option that applies) a. S1 S2 b. S1 S2 S3 c. Murmur d. Pericardial friction rub. 328- The healthcare provider prescribes ceftazidime (Fortax) 35 mg every 8 hours IM for an infant. The 500 mg vial is labeled with the instruction to add 5.3 ml diluent to provide a concentration of 100 mg/ml. How many ml should the nurse administered for each dose? (Enter numeric value only. If rounding is required, round to the nearest tenth) a. 0.4 Rationale: 35mg/100mg x 1 = 0.35 = 0.4 ml 329- The nurse notes that a client has been receiving hydromorphone (Dilaudid) every six hours for four days. What assessment is most important for the nurse to complete?a. Auscultate the client's bowel sounds b. Observe for edema around the ankles c. Measure the client‟s capillary glucose level d. Count the apical and radial pulses simultaneously 330- A female client is admitted with end stage pulmonary disease is alert, oriented, and complaining of shortness of breath. The client tells the nurse that she wants “no heroic measures” taken if she stops breathing, and she asks the nurse to document this in her medical record. What action should the nurse implement? a. Ask the client to discuss “do not resuscitate” with her healthcare provider b. Provide supplemental oxygen per venturi mask at 24 % concentration c. Allow the client‟s family to stay at the client‟s bedside. d. Notify the healthcare provider the client‟s respiratory status. 331- A client is receiving a full strength continuous enteral tube feeding at 50 ml/hour and has developed diarrhea. The client has a new prescription to change the feeding to half strength. What intervention should the nurse implement? a. Add equal amounts of water and feeding to a feeding bag and infuse at 50ml/hour b. Continue the full strength feeding after decreasing the rate of infusion to 25 ml/hr. c. Maintain the present feeding until diarrhea subsides and the begin the next new prescription. d. Withhold any further feeding until clarifying the prescription with healthcare provides. 332- A female client reports that her hair is becoming coarse and breaking off, that the outer part of her eyebrows have disappeared, and that her eyes are all puffy. Which follow-up question is best for the nurse to ask? a. “Is there a history of female baldness in your family?” b. “Are you under any unusual stress at home or work?” c. “Do you work with hazardous chemicals?” d. “Have you noticed any changes in your fingernails?” 333- After a third hospitalization 6 months ago, a client is admitted to the hospital with ascites and malnutrition. The client is drowsy but responding to verbal stimuli and reports recently spitting up blood. What assessment finding warrants immediate intervention by the nurse? a. Bruises on arms and legs b. Round and tight abdomen c. Pitting edema in lower legs d. Capillary refill of 8 seconds 334- After the nurse witnesses a preoperative client sign the surgical consent form, the nurse signs the form as a witness. What are the legal implications of the nurse‟s signature on the client‟s surgical consent form? (Select all that apply) a. The client voluntarily grants permission for the procedure to be done b. The surgeon has explained to the client why the surgery is necessary. c. The client is competent to sign the consent without impairment of judgment d. The client understands the risks and benefits associated with the proceduree. After considering alternatives to surgery, the client elects to have the procedure. 335- Following surgery, a male client with antisocial personality disorder frequently requests that a specific nurse be assigned to his care and is belligerent when another nurse is assigned. What action should the charge nurse implement? a. Ask the client to explain why he constantly request the nurse b. Encourage the client to verbalize his feelings about the nurse c. Reassure the client that his request will be met whenever possible. d. Advise the client that assignments are not based on client requests 336- A client with cervical cancer is hospitalized for insertion of a sealed internal cervical radiation implant. While providing care, the nurse finds the radiation implant in the bed. What action should the nurse take? a. Call the radiology department b. Reinsert the implant into the vagina c. Apply double gloves to retrieve the implant for disposal. d. Place the implant in a lead container using long-handled forceps 337- The client with which type of wound is most likely to need immediate intervention by the nurse? a. Laceration b. Abrasion c. Contusion d. Ulceration 338- The nurse is planning care for a client admitted with a diagnosis of pheochromocytoma. Which intervention has the highest priority for inclusion in this client‟s plan of care? a. Record urine output every hour b. Monitor blood pressure frequently c. Evaluate neurological status d. Maintain seizure precautions 339- When caring for a client who has acute respiratory distress syndrome (ARDS), the nurse elevates the head of the bed 30 degrees. What is the reason for this intervention? a. To reduce abdominal pressure on the diaphragm b. to promote retraction of the intercostal accessory muscle of respiration c. to promote bronchodilation and effective airway clearance d. to decrease pressure on the medullary center which stimulates breathing 340- When assessing a mildly obese 35-year-old female client, the nurse is unable to locate the gallbladder when palpating below the liver margin at the lateral border of the rectus abdominal muscle. What is the most likely explanation for failure to locate the gallbladder by palpation? a. The client is too obese b. Palpating in the wrong abdominal quadrant c. The gallbladder is normald. Deeper palpation technique is needed 341- A woman with an anxiety disorder calls her obstetrician‟s office and tells the nurse of increased anxiety since the normal vaginal delivery of her son three weeks ago. Since she is breastfeeding, she stopped taking her antianxiety medications, but thinks she may need to start taking them again because of her increased anxiety. What response is best for the nurse to provide this woman? a. Describe the transmission of drugs to the infant through breast milk b. Encourage her to use stress relieving alternatives, such as deep breathing exercises c. Inform her that some antianxiety medications are safe to take while breastfeeding d. Explain that anxiety is a normal response for the mother of a 3-week-old. 342- An older male client with a history of type 1 diabetes has not felt well the past few days and arrives at the clinic with abdominal cramping and vomiting. He is lethargic, moderately, confused, and cannot remember when he took his last dose of insulin or ate last. What action should the nurse implement first? a. obtain a serum potassium level b. administer the client's usual dose of insulin c. assess pupillary response to light d. Start an intravenous (IV) infusion of normal saline 343- A client who received multiple antihypertensive medications experiences syncope due to a drop in blood pressure to 70/40. What is the rationale for the nurse‟s decision to hold the client‟s scheduled antihypertensive medication? a. Increased urinary clearance of the multiple medications has produced diuresis and lowered the blood pressure b. The antagonistic interaction among the various blood pressure medications has reduced their effectiveness c. The additive effect of multiple medications has caused the blood pressure to drop too low. d. The synergistic effect of the multiple medications has resulted in drug toxicity and resulting hypotension. 344- Which client is at the greatest risk for developing delirium? a. An adult client who cannot sleep due to constant pain. b. an older client who attempted 1 month ago c. a young adult who takes antipsychotic medications twice a day d. a middle-aged woman who uses a tank for supplemental oxygen 345- Which intervention should the nurse include in a long-term plan of care for a client with Chronic Obstructive Pulmonary Disease (COPD)? a. Reduce risks factors for infection b. Administer high flow oxygen during sleep c. Limit fluid intake to reduce secretions d. Use diaphragmatic breathing to achieve better exhalation346- Which location should the nurse choose as the best for beginning a screening program for hypothyroidism? a. A business and professional women's group. b. An African-American senior citizens center c. A daycare center in a Hispanic neighborhood d. An after-school center for Native-American teens 347- A female client has been taking a high dose of prednisone, a corticosteroid, for several months. After stopping the medication abruptly, the client reports feeling “very tired”. Which nursing intervention is most important for the nurse to implement? a. Measure vital signs b. Auscultate breath sounds c. Palpate the abdomen d. Observe the skin for bruising 348- A male client reports the onset of numbness and tingling in his fingers and around his mouth. Which lab is important for the nurse to review before contacting the health care provider? a. capillary glucose b. urine specific gravity c. Serum calcium d. white blood cell count 349- What explanation is best for the nurse to provide a client who asks the purpose of using the log-rolling technique for turning? a. working together can decrease the risk for back injury b. The technique is intended to maintain straight spinal alignment. c. Using two or three people increases client safety. d. turning instead of pulling reduces the likelihood of skin damage 350- A client receiving chemotherapy has severe neutropenia. Which snack is best for the nurse to recommend to the client? a. Plain yogurt with sweetened with raw honey b. Peanuts in the shell, roasted or un-roasted. c. Aged farmer‟s cheese with celery sticks d. Baked apples topped with dried raisins 351- Which action should the school nurse take first when conducting a screening for scoliosis? a. Compare dorsal measurement of trunk b. Extend arms over head for visualization c. Inspect for symmetrical shoulder height. d. Observe weight-bearing on each leg. 352- An unlicensed assistive personnel (UAP) assigned to obtain client vital signs reports to the charge nurse that a client has a weak pulse with a rate of 44 beat/ minutes. What action should the charge nurse implement?a- Instruct the UAP to count the client apical pulse rate for sixty seconds b- Determine if the UAP also measured the client‟s capillary refill time. c- Assign a practical nurse (LPN) to determine if an apical radial deficit is present. d- Notify the health care provider of the abnormal pulse rate and pulse volume. 353- After a sudden loss of consciousness, a female client is taken to the ED and initial assessment indicate that her blood glucose level is critically low. Once her glucose level is stabilized, the client reports that was recently diagnosed with anorexia nervosa and is being treated at an outpatient clinic. Which intervention is more important to include in this client‟s discharge plan? a. Describe the signs and symptoms of hypoglycemia. b. Encourage a low-carbohydrate and high-protein diet c. Reinforce the need to continue outpatient treatment d. Suggest wearing a medical alert bracelet at all time. 354- A client with a peripherally inserted central catheter (PICC) line has a fever. What client assessment is most important for the nurse to perform? a- Observe the antecubital fossa for inflammation. b- Encourage a low-carbohydrate and high-protein diet c- Reinforce the need to continue outpatient treatment d- Suggest wearing a medical alert bracelet at all time. 355- The nurse administers an antibiotic to a client with respiratory tract infection. To evaluate the medication‟s effectiveness, which laboratory values should the nurse monitor? Select all that apply a- White blood cell (WBC) count b- Sputum culture and sensitivity c- droplet precautions d- protective environment 356- A client is admitted to isolation with the diagnosis of active tuberculosis. Which infection control measures should the nurse implement? a- Negative pressure environment b- contact precautions c- droplet precautions d- protective environment 357- A school nurse is called to the soccer field because a child has a nose bleed (epistaxis). In what position should the nurse place the child? a- Sitting up and leaning forward b- Rented movies and borrowed books to use while passing time at home c- Altered consciousness within the first 24 hours after injury.358- A young adult who is hit with a baseball bat on the temporal area of the left skull is conscious when admitted to the ED and is transferred to the Neurological Unit to be monitored for signs of closed head injury. Which assessment finding is indicative of a developing epidural hematoma? a- Altered consciousness within the first 24 hours after injury. b- contact precautions c- droplet precautions d- protective environment 359- A female client with breast cancer who completed her first chemotherapy treatment today at an out-patient center is preparing for discharge. Which behavior indicates that the client understands her care needs a- Rented movies and borrowed books to use while passing time at home b- Sputum culture and sensitivity b- droplet precautions c- protective environment 360- Which instruction should the nurse provide a pregnant client who is complaining of heartburn? a- Limit fluids between meals to avoid over distension of the stomach b- Take an antacid at bedtime and whenever symptoms worsen c- Maintain a sitting position for two hours after eating. d- Eat small meal throughout the day to avoid a full stomach. 361- A client is admitted to the intensive care unit with diabetes insipidus due to a pituitary gland tumor. Which potential complication should the nurse monitor closely? a- Hypokalemia b- Ketonuria. c- Peripheral edema d- Elevated blood pressure 362- A female client reports she has not had a bowel movement for 3 days, but now is defecating frequent small amount of liquid stool. Which action should the nurse implement? a- Digitally check the client for a fecal impaction b- Bilateral Wheezing. c- Peripheral edema d- Elevated blood pressure 363- After changing to a new brand of laundry detergent, an adult male reports that he has a fine itchy rash. Which assessment finding warrants immediate intervention by the nurse? a- Bilateral Wheezing. b- Ketonuria. c- Peripheral edema d- Elevated blood pressure 364- The nurse should teach the parents of a 6 year-old recently diagnosed with asthma that the symptom of acute episode of asthma are due to which physiological response? a- Inflammation of the mucous membrane & bronchospasm b- Ketonuria. c- Peripheral edema d- Elevated blood pressure 365- A 10-year-old who has terminal brain cancer asks the nurse, "What will happen to my body when I die?" How should the nurse respond? a- "The heart will stop beating & you will stop breathing." b- Ketonuria. c- Peripheral edema d- Elevated blood pressure 366- The nurse is assessing a 3-month-old infant who had a pylorotomy yesterday. This child should be medicated for pain based on which findings? Select all that apply: a- Restlessness b- Clenched Fist c- Increased pulse rate d- Increased respiratory rate. e- Increased temperature f-Peripheral pallor of the skin 367- The nurse is preparing to administer an oral antibiotic to a client with unilateral weakness, ptosis, mouth drooping and, aspiration pneumonia. What is the priority nursing assessment that should be done before administering this medication? a- Ask the client about soft foods preferences b- Auscultate the client‟s breath sounds c- Obtain and record the client‟s vital signs e- Determine which side of the body is weak. 368- The nurse who is working on a surgical unit receives change of shift report on a group of clients for the upcoming shift. A client with which condition requires the most immediate attention by the nurse? a- Gunshot wound three hours ago with dark drainage of 2 cm noted on the dressing. b- Mastectomy 2 days ago with 50 ml bloody drainage noted in the Jackson-pratt drain.c- Collapsed lung after a fall 8h ago with 100 ml blood in the chest tube collection container d- Abdominal-perineal resection 2 days ago with no drainage on dressing who has fever and chills. 369- The nurse is caring for a client who had gastric bypass surgery yesterday. Which intervention is most important for the nurse to implement during the first 24 postoperative hours? a- Insert an indwelling urinary catheter b- Monitor for the appearance of an incisional hernia c- Instruct the client to eat small frequent meals d- Measure hourly urinary output. 370- When preparing to discharge a male client who has been hospitalized for an adrenal crisis, the client expresses concern about having another crisis. He tells the nurse that he wants to stay in the hospital a few more days. Which intervention should the nurse implement? a- Administer antianxiety medication prior to providing discharge instructions b- Schedule an appointment for an out-patient psychosocial assessment. c- Obtain a blood cortisol level after last dose of synthetic ACTH d- Encourage the healthcare provider to delay the client‟s discharge. 371- In caring for a client with Cushing syndrome, which serum laboratory value is most important for the nurse to monitor? a- Lactate b- Glucose c- Hemoglobin d- Creatinine 372- Azithromycin is prescribed for an adolescent female who has lower lobe pneumonia and recurrent chlamydia. What information is most important for the nurse to provide to this client? a- Have partner screened for human immunodeficiency virus b- Report a sudden onset arthralgia to the healthcare provider c- Decrease intake of high-fat-foods, caffeine, and alcohol d- Use two forms of contraception while taking this drug. 373- A client in the emergency center demonstrates rapid speech, flight of ideas, and reports sleeping only three hours during the past 48h. Based on these finding, it is most important for the nurse to review the laboratory value for which medication? a- Olanzapine b- Divalproex. c- Lorazepam d-Fluoxetine 374- A male client who is admitted to the mental health unit for treatment of bipolar disorder has a slightly slurred speech pattern and an unsteady gait. Which assessment finding is most important for the nurse to report to the healthcare provider? a- Blood alcohol level of 0.09%b- Serum lithium level of 1.6 mEq/L or mmol/l (SI) c- Six hours of sleep in the past three days. d- Weight loss of 10 pounds (4.5 kg) in past month. 375- A client was admitted to the cardiac observation unit 2 hours ago complaining of chest pain. On admission, the client‟s EKG showed bradycardia, ST depression, but no ventricular ectopy. The client suddenly reports a sharp increase in pain, telling the nurse, “I feel like an elephant just stepped on my chest” The EKG now shows Q waves and ST segment elevations in the anterior leads. What intervention should the nurse perform? a- Increase the peripheral IV flow rate to 175 ml/hr to prevent hypotension and shock b- Administer prescribed morphine sulfate IV and provide oxygen at 2 L/min per nasal cannula. c- Obtain a stat 12 lead EKG and perform a venipuncture to check cardiac enzymes levels. d- Notify the healthcare provider of the client‟s increase chest pain a call for the defibrillator crash cart. 376- The nurse is developing a teaching program for the community. What population characteristic is most influential when choosing strategies for implementing a teaching plan? a- Literacy level b- Prevalent learning style c- Median age d- Percent with internet access. 377- A client is being discharged with a prescription for warfarin (Coumadin). What instruction should the nurse provide this client regarding diet? a- Eat approximated the same amount of leafy green vegetables daily so the amount of vitamin K consumed is consistent. b- Avoid eating all foods that contain any vitamin K because it is an antagonist of Coumadin. c- Increase the intake of dark green leafy vegetables while taking Coumadin d- Eat two servings of raw dark green leafy vegetables daily and continue for 30 days after Coumadin therapy is completed. 378- A client who had a small bowel resection acquired methicillin resistant staphylococcus aureus (MRSA) while hospitalized. He treated and released, but is readmitted today because of diarrhea and dehydration. It is most important for the nurse to implement which intervention. a- Maintain contact transmission precaution b- Review white blood cell (WBC) count daily c- Instruct visitors to gown and wash hands d- Collect serial stool specimens for culture 379- A postoperative female client has a prescription for morphine sulfate 10 mg IV q3 hours for pain. One dose of morphine was administered when the client was admitted to the post anesthesia care unit (PACU) and 3 hours later, the client is again complaining of pain. Her current respiratory rate is 8 breaths/minute. What action should the nurse take? a- Provide oxygen 100% via facemask b- Check peripheral tendon reflexes c- Give another IV dose of morphine d- Administer Naxolone IV 380- Which intervention is most important for the nurse to include in the plan of care for an older woman with osteoporosis? a- Evaluate the client‟s orientation to time and place b- Place the client on fall precautions c- Encourage the client to drink milk with meals d-Assess the client‟s breath sounds daily. 381- Based on the information provided in this client‟s medical record during labor, which should the nurse implement? (Click on each chart tab for additional information. Please be sure to scroll to the bottom right corner of each tab to view all information contained in the client‟s medical record) a- Apply oxygen 10 l/mask b- Stop the oxytocin infusion c- Turn the client to the right lateral position. d- Continue to monitor the progress of labor. 382- An unlicensed assistive personnel UAP leaves the unit without notifying the staff. In what order should the unit manager implement this intervention to address the UAPs behavior? (Place the action in order from first on top to last on bottom.) 1. Note date and time of the behavior. 2. Discuss the issue privately with the UAP. 3. Plan for scheduled break times. 4. Evaluate the UAP for signs of improvement. 383- A client with intestinal obstructions has a nasogastric tube to low intermittent suction and is receiving an IV of lactated Ringer‟s at 100 ml/H. which finding is most important for the nurse to report to the healthcare provider? a- Gastric output of 900 mL in the last 24 hours b- Serum potassium level of 3.1 mEq/L or mmol/L (SI) c- Increased blood urea nitrogen (BUN) d- 24-hour intake at the current infusion rate. 384- Which type of Leukocyte is involved with allergic responses and the destruction of parasitic worms? a- Neutrophils b- Lymphocytes c- Eosinophils d- Monocytes385- Several months after a foot injury, and adult woman is diagnosed with neuropathic pain. The client describes the pain as severe and burning and is unable to put weight on her foot. She asks the nurse when the pain will “finally go away.” How should the nurse respond? a- Explain the healing from injury can take many months b- Assist the client in developing a goal of managing the pain. c- Encourage the client to verbalize her fears about the pain d- Complete an assessment of the client‟s functional ability. 386- One day following an open reduction and internal fixation of a compound fracture of the leg, a male client complains of “a tingly sensation” in his left foot. The nurse determines the client‟s left pedal pulses are diminished. Based on these finding, what is the client‟s greatest risk? a- Reduce pulmonary ventilation and oxygenation related to fat embolism. b- Neurovascular and circulation compromise related to compartment syndrome. c- Wound infection and delayed healing to fractured bone protrusion. d- Venous stasis and thrombophlebitis related to postoperative immobility. 387- The nurse is completing a head to be assessment for a client admitted for observation after falling out of a tree. Which finding warrants immediate intervention by the nurse? a- Sluggish pupillary response to light b- Clear fluid leaking from the nose. c- Complaint of severe headache d- Periorbital ecchymosis of right eye. 388- Following the evacuation of a subdural hematoma, an older adult develops an infection. The client is transferred to the neuro intensive care unit with a temperature of 101.8 F (39.3 C) axillary, pulse of 180 beats/minute, and a blood pressure of 90/60 mmHg. What is the priority intervention to include in this client‟s plan of care? A. Confusion B. Check neuro vital sings every 4 hours. C. Maintain intravenous access. D. Keep the suture line clean and dry. 389- The nurse plans to collect a 24-hour urine specimen for a creatinine clearance test. Which instruction should the nurse provide to the adult male client? a-Urinate at the specified time, discard this urine, and collect all subsequent urine the next 24 hours. b- Cleanse around the meatus, discard first portion of voiding, and collect the rest in a sterile bottle. c- For the next 24 hours, notify nurse when the bladder is full, and the nurse will collect catheterized specimens. d-Urinate immediately into a urinal, and the lab will collect the specimen every 6 hours, for the next 24 hours. 390- The family of an older woman reports that they are no longer capable of caring for her at home. While performing the admission assessment at long-term care facility, the nurse determines that the client is incontinent of urine, has dry mucous membranes and has a large bruise on the coccyx. Which interventions should the nurse include in the plan of care? (Select all that apply.) a- Thicken liquids and provide pureed foods. b- Offer beverages at frequent interval. c- Apply a barrier cream to perianal areas. d- Implement toileting program. e- Report suspicion of the elder abuse. 391- The nurse is measuring the output of an infant admitted for vomiting and diarrhea. During a 12 hour shift, the infant drink 4 ounces of Pedialyte. Vomits 25 ml, and voids twice. The dry diaper weight 105 grams. Which computer documentation should the nurse enter in the infant‟s record? A. Subtract vomitus from 120 ml Pedialyte them document 95 ml oral intake. B. Compare the difference between the infant‟s body weight and admission weight. C. Document on the flow sheet that the infant voided times 2 and vomited 25 ml. D. Calculate difference in wet and dry diapers and document 80 ml urine. 392- An adult client with a broken femur is transferred to the medical surgical unit to await surgical internal fixation after the application of an external traction device to stabilize the leg. An hour after an opioid analgesic was administered, the client report muscle spasms and pain at the fracture site. While waiting for the client to be transported to surgery, which action should the nurse implement!? A- Observe for signs of deep vein thrombosis. B- Administer PRN dose of a muscle relaxant. C- Check client´s most recent electrolyte values. D- Reduce the weight on the traction device. 393- Which client´s vital signs indicating increased intracranial pressure (IPC) should the nurse report to the healthcare provider? A- P 70, BP 120/60 mmHg; P 100, BP 90/60 mmHg; rapid respiration. B- P 55, BP 160/70 mmHg, P 50, BP 194/70 mmHg, irregular respirations. C- P 130, BP 190/90 mmHg, P 136, BP 200/100 mmHg, Kussmaul respirations. D- P 110, BP 130/70 mmHg, P 100, BP 110/70 mmHg, shallow respirations. 394- The nurse enters the room of a client who is awaiting surgery for appendicitis. The unlicensed assistive personnel (UAP) has a helped the client to a position of comfort with the right leg flexed and has applied a heating pad to the client´s abdomen to relieve the client´s pain. Which action should the nurse implement first? A- Remove the heating pad from the client´s abdominal area. B- Determine if the consent form has been signed by the client. C- Evaluate the effectiveness of the heating pad in relieving pain. D- Confirm that the UAP has assisted the client to a position of comfort. 395- The nurse is inspecting the thoracic region of an adolescent client. Which finding is consistent with scoliosis? A- Asymmetry. B- Barrel shape. C- Erythema. D- Masses. 396- When preparing a client for discharge from the hospital following a cystectomy and urinary diversion to treat bladder cancer, which instruction is most important for the nurse to include in the client´s discharge teaching plan? A- Report any signs of cloudy urine output. B- Seek counseling for body image concerns. C- Frequently empty bladder to avoid distension. D- Follow instructions for self-care toileting. 397- A client a traumatic brain injury becomes progressively less responsive to stimuli. The client has a “Do Not Resuscitate” prescription, and the nurse observes that the unlicensed assistive personnel (UAP) has stopped turning the client from side as previously scheduled. What action should the nurse take? A- Encourage the UAP to provide comfort care measures only. B- Assume total care of the client to monitor neurologic function. C- Advise the UAP to resume positioning the client on schedule. D- Assign a practical nurse to assist the UAP in turning the client. 398- While caring for a client´s postoperative dressing, the nurse observes purulent wound drainage. Previously, the wound was inflamed and tender but without drainage. Which is the most important action for the nurse to take: A- Monitor the client´s while blood cell count (WBC). B- Determine If the drainage has an unpleasant odor. C- Request a culture and sensitivity of the wound. D-Cleanse the wound with a saline solution. 399- In assessing a client with diabetes mellitus Type 1, the nurse that the client´s respirations have changed from 16 with a normal depth to 32 and deep, and the client has become lethargic. What assessment data should the nurse obtain next? a- Core temperature. b- Blood glucose. c- Arterial blood gases. d- D- Oxygen saturation. 400- When caring for a client who has acute respiratory distress syndrome (ARDS), the elevates the head of bed 30 degrees. What is the reason for this intervention? a- To promote retraction of the intercostal accessory muscles. b- To reduce abdominal pressure on the diaphragm. c- To decrease pressure on the medullary center which stimulates breathing. d- To promote bronchodilation and effective airway clearance. 401- While assisting a client who recently had a hip replacement onto the bed pan, the nurse notices that there is a small amount of bloody drainage on the surgical dressing, the client´s skin is warm to the touch, and there is a strong odor from the urine. Which action should the nurse take? A- Insert an indwelling urinary catheter. B- Obtain a urine sample from the bed pan. C- Remove dressing and assess surgical site. D- Measure the client´s oral temperature. 402- A male client with heart failure (HF) reports heart palpations and difficulty breathing when Lying flat. When he requests additional pillows, the nurse determines that his heart rate is 125 beats/ minute, and respiratory rate is 22 breaths/ minute. Which intervention is most important for the nurse to implement? A- Listen to S1 and S2 heart sounds. B- Obtain manual blood pressure. C- Auscultate bilateral lung fields. D- Palpate the peripheral pulses. / 403- The nurse is teaching a male client who recently diagnosed with gout. Which statement indicates that the client understands how to manage his gout? A- During an acute attack, fluid intake should be increased to at least 3 liters daily. B- Female offspring have a high risk for developing gout in adulthood. C- Foods that are high in fast and carbohydrates should be avoided. D- Obesity has many health care consequences but is unrelated to managing gout. 404- A client with a new diagnosis of Raynaud´s disease lives alone. Which instruction should the nurse include in this client´s discharge teaching plan? A- Wear TED stockings at night. B- Hire a care-giver for 8 hours daily. C- Keep room temperature 80 F. D- Develop a walking exercise routine. 405- A client who is admitted to the medical unit with fluid volume overload has 2+ dependent edema in the lower extremities and a serum sodium level of 115 mEq/L (115MMOl/L). Which assessment should the nurse complete next? A- Skin turgor. B- Bowel sounds. C- Mental status. D- Urinary output. 406- A 6-year-old child who had surgery yesterday absolutely refuses to use the incentive spirometer. Which intervention should the nurse implement? A- Ask the mother to assist when it is time to use the spirometer. B- Contract with the child to use spirometer only after meals. C- Allow child to choose when to perform incentive spirometry. D- Blow out lights, blow bubbles, and encourage child´s laughing. 407- A client with arthritis has been receiving treatment with naproxen and now reports ongoing stomach pain; increasing weakness; and fatigue. Which laboratory test should the nurse stomach pain, increasing weakness, and fatigue. Which laboratory test should the nurse monitor? A- Hemoglobin. B- Serum calcium. C- Osmolality. D- Erythrocyte sedimentation rate. 408- A client with hyperthyroidism is receiving propranolol. Which finding indicates that the medication is having the desired effect? A- Decrease in serum T4 levels. B- Goiter no longer palpable. C- Decrease in pulse rate. D- Increase in blood pressure. 409- When is it most important for the nurse to assess a pregnant client´s deep tendon reflexes (DTRs)? A- Within the first trimester of pregnancy. B- If the client has an elevated blood pressure. C- When the client has ankle edema. D- During admission to labor and delivery. 410- When conducting an admission assessment of a newborn infant, the nurse identifies a loud, harsh heart murmur. The healthcare provider prescribes an echocardiogram. What action is most important for the nurse to include in the infant´s plan care? A- Compare daily weight changes. B- Monitor for signs of heart failure. C- Offer bottled formula between breast feedings. D- Keep the head of the infant´s crib elevated.***NOT SURE*** 411- A client with rapid respirations and audible rhonchi is admitted to the intensive care unit because of a pulmonary embolism (PE). Low-flow oxygen by nasal cannula and weight based heparin protocol is initiated. Which intervention is most important for the nurse to include in this client´s plan of care? A- Evaluate daily blood clothing factors. B- Monitor deep vein blood flow using Doppler. C- Maintain strict bed rest. D- Apply antiembolism stockings. 412- One year after being discharge from the burn trauma unit, a client with a history of 40 percent full thickness burns is admitted with bone pain and muscle weakness. Which intervention is should the nurse include in this client´s plan of care? A- Encourage progressive active range of motion. B- Instruct the client in the use of muscle strengthening exercises. C- Teach need for dietary and supplemental vitamin D3. D- Explain the need for skin exposure to sunlight without sunscreen. 413- The nurse identifies an electrolyte imbalance, an elevated pulse rate, and an elevated blood pressure for a client with chronic kidney disease. Which is the most important action for the nurse to take? A- Record usual eating patterns. B- Monitor daily sodium intake. C- Measure ankle circumference. D- Auscultate for irregular heart rate. 414- A nurse determines that more than 25% of the students at a middle school are overweight. The nurse presents the information at a parent-teacher meeting. What action is most important for the nurse to include in the meeting? A- Provide information on ways to increase activity for the family. B- Have several teachers talk about health risks associated with obesity. C- Determine the parent‟s degree of concern about their children´s weight. D- Distribute a shopping list of suggested healthy snack items. 415- The nurse is completing the admission assessment of s 3-year-old who is admitted with bacterial meningitis and hydrocephalus. Which assessment finding is evidence that the child is experiencing increased intracranial pressure (ICP)? A- Blood pressure fluctuations and syncope. B- Tachycardia and tachypnea. C- Sluggish and unequal pupillary responses. D- Increased head circumference and bulging fontanels. 416- While assessing an older client´s fall risk, the client tells the nurse that they live at home alone and has never fallen. What action should the nurse take? A- Inform the client falls occur more often in the hospital than at home. B- Record a minimal risk for falls, documenting the client´s statement. C- Place the client on a high fall risk protocol because of his age. D- Continue to obtain the client data needed to complete the fall risk survey. 417- A client is admitted with acute pancreatitis. The client admits to drinking a pint of bourbon daily. The nurse medicates the client for pain and monitors vital signs every 2 hours. Which finding should the nurse report immediately to the healthcare provider? A- Yellowing and itching of skin. B- Confusion and tremors.C- Abdominal pain and vomiting. D- Anorexia and abdominal distention. 418- An older client with a 3day history of abdominal distention is admitted with a small bowel obstruction. The nurse inserts a nasogastric tube and attaches it to low intermittent suction. Which ongoing client assessment takes priority when providing care? A- Auscultate bowel sounds. B- Observe skin integrity. C- Monitor fluid balance. D- Measure abdominal girth. 419- The nurse is completing an admission assessment for a male client with paranoid schizophrenia. The client tells the nurse that the staff dislikes him. What action should the nurse take? A- Ask the client if he has a plan to harm himself. B- Determine if the client has formulated any plans regarding the staff. C- Observe the client for obsessive activities such as repeated hand washing. D- Assess the client´s speech pattern for a flight of ideas. 420- After spending two weeks in the intensive care unit being treated for severe burns, a client is transferred to a progressive care unit with extensive dressings, a large healing wound, and a central venous access catheter. Which intervention should the nurse include in the plan of care? A- Perform frequent passive range of motion. B- Enforce nothing by mouth (NPO). C- Encourage expression of anger and emotions. D- Assist with daily progressive ambulation. 421- A client is receiving ophthalmic drops preoperatively for a cataract extraction and asks the nurse why the healthcare provider has prescribed all these medications. Which information should the nurse included when responding to this client? (Select all that apply.) A- One of the medications is used to anesthetize the corneal surface. B- Pupillary dilation is necessary to access the eye chamber for lens removal. C- The iris must be paralyzed during surgery to prevent it from reacting to light. D- A medication is used to induce sleep during the procedure. E- These medications assist in obstructing client´s vision during the surgery. 422- The nurse is ready to insert an indwelling urinary catheter as seen in the picture. At this point in the procedure, what actions should the nurse take before inserting the catheter? (Select all that apply) Ask the client to bear down as if voiding to relax the sphincter B. Complete perianal care with soap and water C. Gently palpate the client‟s bladder for distention D. Hold the catheter 3 – 4 inches (7.5 – 10 cm) from its tip E. Secure the urinary drainage bag to the bed frame422- In evaluating the effectiveness of a postoperative client‟s intermittent pneumatic compression devices, which assessment is most important for the nurse to complete? Ans: Observe both lower extremities for redness and swelling 423- The nurse working in the psychiatric clinic has phone messages from several clients. Which call should the nurse return first? ANS: A family member of a client with dementia who has been missing for five hours. 424- An adult client experiences a gasoline tank fire when riding a motorcycle and is admitted to the emergency department (ED) with full thickness burns to all surfaces of both lower extremeties. What percentage of body surface area should the nurse document in the electronic medical record (EMR)? ANS: 36% 425- The nurse is assessing a Vietnamese client who has terminal lung cancer when a family member comes to visit and brings ethnic food from home for the client. The client is on a soft mechanical regular diet. What intervention should the nurse implement? ANS: Allow food from home that is included in a mechanical soft diet. One day after abdominal surgery, an obese client complains of pain and heaviness in the right calf. What action should the nurse implement? Observe for unilateral swelling • The healthcare provider prescribes a low-fiber diet for a client with ulcerative colitis. Which food selection would indicate to the nurse the client understands they prescribed diet?• Roasted turkey canned vegetables • Baked potatoes with skin raw carrots • Pancakes whole-grain cereal's • Roast pork fresh strawberries • To prevent infection by auto contamination during the acute phase of recovery from multiple burns, which intervention is most important for the nurse to implement? • Dress each wound separately. • Avoid sharing equipment between multiple clients. • Use gown, mask and gloves with dressing change. • Implement protective isolation. • The nurse is teaching a client how to perform colostomy irrigations. When observing the client‟s return demonstration, which action indicated that the client understood the teaching? • Keeps the irrigating container less than 18 inches above the stom [Show More]

Last updated: 1 year ago

Preview 1 out of 83 pages

Reviews( 0 )

$12.00

Add to cart

Instant download

Can't find what you want? Try our AI powered Search

OR

GET ASSIGNMENT HELP
58
0

Document information


Connected school, study & course


About the document


Uploaded On

Jun 16, 2021

Number of pages

83

Written in

Seller


seller-icon
QuizMaster

Member since 3 years

11 Documents Sold


Additional information

This document has been written for:

Uploaded

Jun 16, 2021

Downloads

 0

Views

 58

Document Keyword Tags

Recommended For You


$12.00
What is Browsegrades

In Browsegrades, a student can earn by offering help to other student. Students can help other students with materials by upploading their notes and earn money.

We are here to help

We're available through e-mail, Twitter, Facebook, and live chat.
 FAQ
 Questions? Leave a message!

Follow us on
 Twitter

Copyright © Browsegrades · High quality services·